M1539856502esolutions ICSE Geography Worksheet

You might also like

Download as pdf or txt
Download as pdf or txt
You are on page 1of 72

DRAMA

CHAPTER

INTERPRETATION OF TOPO-GRAPHICAL MAPS


11
 WORKSHEET-1
Ans.1 (a) (i) The highest point- 078065 (.365)
(ii) Surveyed tree north of Rampura Khera- 072074.
(b) (i) Butri- Left Bank
(ii) Padrugarh- Right Bank.
(c) (i) 0704- Trellised
(ii) 0705- Radial.
(d) Presence of seasonal streams, lined perennial well, cart track motorable in the dry season/ dry tanks/
dry river beds. [Any two]
(e) The distance between the settlement Bhatana and Makawal along the cart track on the map is
9.8 cms
Scale- 2 cm = 1 km
Therefore, ½ x 9.8 = 4.9 kms distance on the ground.
(f) (i) Dattani from Marol- South West.
(ii) Dhavli from Makawal- North East.
(g) (i) Monthly fair at Marol. 
(ii) Agriculture/Farming. 
(h) (i) The black broken line in 0807- Disappearing drainage pattern.
(ii) The vertical black line close to Easting 10- Longitude line- 72°35’ E.
(i) Presence of Lined perennial wells all around the settlement.
(j) (i) 0905- Broken ground, seasonal streams.
(ii) 1003- PO, Temple, cart track, seasonal tank, Lined perennial well. [Any one]
Ans.2 (a) (i) Spot height .542-977827
(ii) Temple at Juvol- 926828
(b) (i) R.F stands for Representative Fraction. It is the ratio between the distance on the map to the
distance on the corresponding ground and is in fraction. The numerator denotes the length on
the map and the denominator denotes the actual distance on the ground. E.g. 1:50,000.
(ii) Eastings 91 to 01 = 10 grids and Northings 73 to 83 = 10 grids
Each grid = 1 sq.km (scale given 2 cm = 1 km)
10 x 10 = 100 sq. kms.
(c) The distance between Juvol and Arniwada on the map is 5 cms
Scale given 2 cm = 1 km
½ x 5= 2.5 kms distance between the Juvol and Arniwada on the ground.
(d) (i) East to West. 
(ii) Right Bank. 
(e) (i) Well irrigation is the main form of irrigation.
(ii) 1) Since this area in G.S. 9478 is a sandy region, the water is absorbed in the sand.
2) Due to high temperature the water evaporates and do not join the river. [Any one]
(f) (i) Ridge/ Col.
(ii) 4R represents the relative depth of the lined perennial well. 
(g) (i) 9576 -Nucleated or Compact.
9774 - Scattered or Dispersed. 
9574- Radial drainage pattern. 
9382- Trellis drainage pattern.

S O L U T I ON S P-1
(h) (i) North East.
(ii) Chitrasani is well connected by metalled road and railway and has good public utility facilities
like PO, RS. 
(i) (i) Man made feature- Temple and Lined Perennial Well.
Natural feature- Ridge/Col. 
(ii) Broken Ground. 
(j) (i) Cart Track and Metalled Road. 
(ii) Agriculture due to the presence of large number of Lined Perennial Wells. 
Ans.3 (a) (i) Surveyed tree 219 north east of Pirojpura settlement- 979756.
(ii) ∆364 in the southern part of the map extract- 957744.
(b) Direction of flow of Banasriver is from north east to west.
Evidence is decreasing spot height from 233 in 9681 to 177 in 9380.
(c) (i) It is the relative height of sand dune which is 12 m.
(ii) It is contour line showing 180 m. above mean sea level.
(d) Area of the region between 93 and 99 eastings and 76 and 81 northings is 30 km² (Area of one
square is 1 km²). There are thirty squares within this region.
(e) (i) North West.
(ii) 542-198=344 mt.
(f) (i) It is a perennial water channel in the seasonal stream.
(ii) It is sand dune.
(g) (i) 9782- Radial drainage pattern.
(ii) 9478- Disappearing drainage pattern.
(h) The region has hot and dry climate with seasonal scanty rainfall.
Evidence is presence of seasonal streams, lined perennial well, broken ground, etc.
(i) Manmade feature- cart track, temple, lined perennial, permanent hut.
Natural feature- broken ground, seasonal river.
(j) (i) Black vertical line is the line of longitude.
(ii) RS denotes railway station.
 [ICSE Marking Scheme 2016]
ll
 WORKSHEET-2
Ans.1 (a) (i) Hamirpura- 0123. 
(ii) Temple – 078187. 
(b) (i) Perennial lined well. 
(ii) Towards Northwest. 
(c) Difference between:
(i) Settlements in 0725- Scattered or Dispersed. 
Settlement Idarla- Compact or Nucleated. 
(ii) Drainage pattern in 0624- Trellised. 
Drainage pattern in 0824- Dendritic. 
(d)  300 m above mean sea level is the value of the contour line in grid square 0226 Contour Interval- 20 m.
(e) (i) Occupation is agriculture.
(ii) Lined perennial well is the main source of water.
(iii) Mode of transport is cart track. [Any two points]
(f) (i) Broken ground is formed due to the result of weathering caused by alternate wet and dry
periods which means that the rainfall is seasonal.
(ii) The cart track which is motorable only during the dry season. 

P-2 G E OG R A P H Y - X
(g) (i) Two natural features in 0527- Seasonal streams, knoll, barren ground.
(ii) Two manmade features in 0325- Cart track, permanent hut, cultivated land.
(h) Dantrai is the most important settlement because of the presence of Post Office, police chowki, high
density of population due to large number of settlements. 
(i) On the south of Northing 21, there 30 grids. Each grid is 1 square kilometres, therefore, total area of
the region is 30 km² or square kilometres. 
(j) (i) The black vertical line is the Line of Longitude. 
(ii) 302 is the spot height (.302 m) above mean sea level. 
Ans.2 (a) 1511, 1711, 1811, 1903.
Dry tank with embankment.
(b) 1606- Radial pattern.
1608- Trellis.
(c) (i) Gautam Maharishi Mandir- 200071.
(ii) .443- 179052.
(d) Vajna (1503)
Two reasons: Presence of a police chowki, Metalled road
(e) (i) West.
(ii) Souh West.
(f) (i) Causeway- A raised road over a small stream but not a bridge/ A raised metalled road over a
non-perennial stream or marshy area or which serves as a temporary bridge.          
(ii) Falls 25 m- It indicates a waterfall which is falling from a height of 25 metres.
(g) (i) 1 Hour.
(ii) 15 sq. kms.
(h) (i) 1610- Seasonal stream, broken ground, dry stream.
(ii) 1903- Permanent settlement, embankment.
(i) Presence of broken ground, causeways, motorable in dry season.
(j) Highest point is 1409 m and contour height is 2402 is 80 m.
Difference is 1409-80= 1129 metres. [ICSE Marking Scheme 2015]
ll
 WORKSHEET-3
Ans.1 (a) (i) ∆217- 940793. 
(ii) Lined well near Chekhla- 925815. 
(b) (i) Balram Nadi.
(ii) Towards North West. 
(c) (i) Broken grounds. 
(ii) Trellised. 
(d) (i) 9573/9680/0076. 
(ii) 9978. 
(e) 9782- Conical Hill. 
9574- Ridge. 
(f) Due to scanty rainfall cultivation is limited in the map extract.
Two reasons-
(i) The northern part of the map is a highland.
(ii) The southern part is covered with sand dunes. 
(g) Antroli from Sangla- North East.
Chekhla from Sangla- North. 

S O L U T I ON S P-3
(h) Seasonal rainfall.
Presence of broken grounds, open scrub, sand dunes, water channel.  [Any one]
(i) The distance between Chitrasani and Pirojpura on the map is 4.6 cms.
Scale given- 2 cm = 1 km
Therefore, ½ x 4.6= 2.3 kms on the ground. 
(j) (i) Scattered. 
(ii) Southern part of the map. 
Ans.2 (a) Malgaon in G.S. 1520. 
(b) Spot height 270 in 145132. 
(c) (i) South East. 
(ii) Cart track and Metalled road. 
(d) (i) 280- Contour height 280m. 
(ii) 281- Spot height 281m. 
(e) In Bamba settlements are nucleated and temporary.
In square 1813 the settlements are dispersed and permanent. 
(f) Four facilities in Anadra- PTO, police chowki, DB (Dak Bunglow), dispensary. 
(g) Telephone. 
(h) (i) Brackish in 1915- It means that water drinking and agricultural purposes.
(ii) Causeway in 1715- A raised road over a small stream but not a bridge/ A raised metalled road
over a non-perennial stream or marshy area or which serves as a temporary bridge.
(iii) 6r in 1618- A relative depth of the bank of the river is 6 m. 
(i) (i) 
Sipu Nadi is in its middle course as the region is a flat level land, there are meanders and the
river is sluggish. 
(ii) The rainfall is seasonal because there are broken grounds, barren patches of land, motorable in
dry season and causeway. 
(j)  The distance in kilometres between the distance stone 20 in 1818 and the causeway in 1715 along the
metalled road on the map = 7 cms.
Scale given- 2 cm = 1 km
½ x 7= 3.5 kms is the distance on the ground.
ll
 WORKSHEET-4
Ans.1 (a) (i) The brick kiln near village Serua- 088131 
(ii) The temple near Asav- 059128 
(b) 0916- Radial. 
0712- Trellis. 
(c) (i) Stony waste- 1014. 
(ii) Open scrub- 0816. 
(d) The distance between the causeways in grid square 0512 and 0808 on the map is 11.4 cms.
Scale – 2 cm = 1 km.
Therefore, ½ x 11.4 = 5.7 kms is the distance along the metalled road. 
(e) (i) Broken ground. 
(ii) It is formed due to the result of weathering caused by alternate wet and dry periods which
means that the rainfall is seasonal. 
(f) (i) Nucleated pattern of settlement. 
(ii) Settlements are closely located. 
(g) General direction of Sipu Nadi is South West. The spot height is decreasing from .261 (1116) to .237
(0408).
(h) In G.S. 0313, the probable occupation is government service in PTO(Post and Telegraph Office) and
Police chauki and in 0413, the occupation is agriculture.
(i) (i) Cart track connects Marol and Mitan. 

P-4 G E OG R A P H Y - X
(ii) Metalled road connects Revdar and Karaunti. 
(j)  The area receives seasonal rainfall, it is evident due to the presence of broken ground, stony waste,
dry streams, open scrubs, causeway. 
Ans.2 (a) (i) Triangulated height 307- 859843. 
(ii) The spot height 196- 835916. 
(b) (i) The confluence of the Sipu River and Mahadeviyo Nala- 8189. 
(ii) Sheet rock- 8088/8188. 
(c) The shortest distance between the temple in grid square 8192 and the perennial lined well at
Bhakodar 8188 on the map is 7 cm. 
Scale- 2 cm = 1 km
Therefore, ½ x 7 = 3.5 kms is the shortest distance on the ground. 
(d) (i) Black curved line in 7788- Broken grounds. 
(ii) The blue line in the bed of the Sipu River- Seasonal stream with water channel. 
(e) (i) The general pattern of settlement is compact or nucleated. 
(ii) Settlements are closely located. 
(f)  The chief form of irrigation is the Lined perennial well. It is necessary because the river and the
rainfall are seasonal. 
(g) (i) Chief mode of transport is Cart track. 
(ii) A large number of red lines are crisscrossing the entire map. 
(h) (i) North-east. 
(ii) South-west. 
(i) (i) Trellis. 
(ii) 25r is the relative depth of the lined perennial well which is 25 metres. 
(j) (i) R.F. stands for Representative Fraction. It is the ratio between the distance on the map to the
distance on the corresponding ground and is in fraction. The numerator denotes the length on
the map and the denominator denotes the actual distance on the ground. E.g. 1:50,000.
(ii) RF is 1 : 50,000.
ll
 WORKSHEET-5
Ans.1 (a) (i) Spot height 261- 1107. 
(ii) Post office- 1003. 
(b) 1606 - Radial. 
1007- Trellised. 
(c) Two natural features in G.S. 0910- Broken grounds and Seasonal dry stream, Dry tank.
(d) The distance from the causeway East of Was to the distance stone 10 on the map is 16 cms.
Scale- 2 cm = 1 km
Therefore, ½ x 16= 8 kms is the distance on the ground. 
(e) (i) Universally accepted scale is Representative Fraction (R.F.) 
(ii) The length of the given map is 10 km (Total 10 grids and each side of grid is 1 km, as per scale 2
cm = 1 km) 
(f) (i) A temple south of Dhavli settlement- 111073. 
(ii) Surveyed tree 277- 177056. 
(g) Cart track and metalled road. 
(h) Agriculture and Forestry. 
(i) Gentle slopes are shown by spacing the contour lines far apart while steep slopes are shown by the
closely placed contour lines. 
(j) (i) 3r in 1103- Relative depth of the dry tank. 
(ii) Open scrub in 1502- It means semi-xerophytic vegetation which grows in seasonal rainfall areas
and is not a cultivable land. It is generally used for cattle rearing. 

S O L U T I ON S P-5
(k) Dattani from Chandela- West of Chandela.
Dhavli from Chandela- North-West of Chandela 
(l)  Two man-made features in grid square 1210- Permanent hut / settlements and Lined perennial well,
cart track. 
(m) Motorable in dry season means that the cart track can be used only in dry season i.e. from September
to May as during rainfall it is flooded with water. It indicates that this region receives scanty and
seasonal rainfall. 
Ans.2 (a) Sunset Point from Anadra- South-East. 
(b) Drainage Pattern in G.S. 2315- Radial. 
(c) Broken ground, Seasonal streams, motorable in dry season, dry tank. 
(d) The distance along the metalled road from the causeway in grid square 1715 to the distance stone
marked 20 in grid square 1818 on the map is 6.2 cm.
Scale- 2 cm = 1 km
½ x 6.2 = 3.1 km is the distance on the ground. 
(e) Representative Fraction is only a fraction and is independent of any particular unit of measurement
and has a universal application. 
(f) (i) Anjini Devi ka Mandir- 229160. 
(ii) ∆1327- 217106. 
(g) Three different kinds of road in G.S. 2411 are Metalled Road, Pack track, Gravelled or unmetalled
road while in G.S. 2515 it is Footpath. 
(h) The two occupations are-
Agriculture as there is yellow coloured region and
Forestry as the entire eastern region is green coloured. 
(i)  The slope in G.S. 2115 is Gentle slope because the contour lines are largely spaced and far apart and
in G.S. 1811, it is Steep slope because the contour lines are closely drawn to each other. 
(j) (i) The fire line in grid square 2316- To prevent the spread of fire in the jungles. 
(ii) The pipeline in grid square 2209- To supply water from the reservoir to Abu. 
(k) (i) The red square in grid square 2514- Temporary Huts. 
(ii) 4r in grid square 1612- Relative height of an embankment which is 4 m from the surrounding
surface. 
(l) Three features seen which attract holiday makers to Abu-
(i) The Dilwara temples, Jaivilas Palace.
(ii) The two lakes- Nakhi Talao and Alwar Talao.
(iii) Sunset Point.
ll
 WORKSHEET-6
Ans.1 (a) (i) ∆364- 957744. 
(ii) The temple at Rampura- 954805. 
(b) Two features that show the region has seasonal rainfall:
(i) Presence of Broken ground and seasonal streams.
(ii) Presence of sand dunes. 
(c) The distance between Juvol (923826) and Arniwada (944817) on the map is 4.8 cm
Scale -2 cm = 1 km
Therefore, ½ x 4.8=2.4 km is the distance on the ground. 
(d) Lined perennial well is the chief form of irrigation. A large number of blue dots can be seen in the
map extract. 
(e) (i) Since this area in G.S. 9478 is a sandy region, the water is absorbed in the sand.
(ii) Due to high temperature the water evaporates and do not join the river.
(iii) This region receives seasonal or low rainfall. [Any two points]

P-6 G E OG R A P H Y - X
(f) (i) Black broken lines in 9575- disappearing stream. 
(ii) Black curved lines in 9879- Broken ground
(g) Two landforms shown by the contours in grid square 9876- Ridge and Watershed. 
(h) (i) The general direction of Balaram Nadi is from South-East to North-West. 
(ii) Balaram Nadi joins on the left bank of the main river. 
(i)  Agriculture is the main occupation of the people in this region as the vast tract of yellow colour
shows cultivable land and the presence of large number of lined perennial wells. 
(j) Scale of the map is the proportion between the distance on the map and the corresponding distance
on the ground.
The scale of the map provided is 2 cm to 1 km. 
ll
 WORKSHEET-7
Ans.1 (a) (i) The temple in village Dhavli- 111073. 
∆480- 137045. 
(b) (i) 6r in grid square 1903- Relative height of embankment which is 6 meters. 
(ii) The word Brackish in grid square 1403- It means well containing salty water which is unfit for
drinking and agriculture. 
(c) (i) It represents a cliff. 
(ii)  Red dotted lines are footpath which is the only means of transport in the mountainous
region. 
(d) The region receives seasonal rainfall. The reasons are the presence of large number of seasonal
streams, broken ground, Open scrub.
(e) Drainage pattern in G.S. 1606 is Radial and in G.S. 1708 is Trellised. 
(f) (i) Pattern of settlement in G.S. 1904- Scattered or Dispersed. 
(ii) Dattani has a Post office. 
(g) Ridge and Saddle. 
(h) The ground distance between the causeway in 1502 and the distance stone marked 14 in grid square
1203 is 7.8 cm
Scale- 2 cm = 1 km
Therefore, ½ x 7.8= 3.9 km is the distance on the ground. 
(i) (i) It is a hilly terrain. 
(ii) It is covered with thick forests. 
(j) (i) Highest spot height-1023
Contour height - 260
Difference in height- 763 m.
(ii) North-West. 
Ans.2 (a) (i) The brown line in grid square 1516 is called Contours.
(ii) The figure written along this line indicates the height in meters of the contour line, i.e. 280 which
is above mean sea level. 
(b) Cart track and Metalled road. 
(c) Gulabganj is on the left bank of Sipu Nadi. It gets its water supply from the Lined perennial well.
(d) Fire line is the space that is kept between the trees approximately, 5 meters apart to prevent forest
fire. It is necessary to avoid the spreading of fire. 
(e) Nature of the canal- Perennial
The length of the canal on the map is 10.6 cm
Scale- 2 cm = 1 km, therefore, ½ x 10.6 = 5.3 km.
(f) 2118- Radial.
2114- Dendritic.

S O L U T I ON S P-7
(g) Anadra is the most important because it has facilities of PTO (Post and Telegraph Office), DB (Dak
Bunglow), Police chauki, Dispensary and is connected with metalled road.
(h) (i) North-East.
(ii) Gulabganj forms a nodal centre which means it is connected with all other villages whereas
Udwariya is not so.
ll
 WORKSHEET-8
Ans.1 (a) (i) ∆592- 928890.
(ii) Temple- 912883. 
(b) (i) South-West. 
(ii) The blue line in Arado N indicate Water Channel. 
(c) The distance between Velavas (868895) and Ranol (883888) on the map is 5 cm.
Scale -2 cm = 1 km
Therefore, ½ x 5= 2.5 km is the distance on the ground. 
(d) (i) 7r indicates Relative depth of the Lined perennial well. 
(ii) Lined perennial wells. 
(e) Seasonal rainfall. 
Presence of broken ground, seasonal dry streams, water channel, Island in the river bed, open scrub,
jungle  [Any one point]
(f)  Contour lines. These are the imaginary lines joining places of same height above the sea level.
(g) Two types of vegetation- Open mixed jungle in G.S. 9187 and Dense mixed jungle in G.S. 9385.
(h) (i) Conical Hill. 
(ii) Island. 
(i) (i) Human settlement in 8989- Linear settlement. 
(ii) Drainage in grid square 9185- Radial. 
(j) Agriculture is the general occupation of people.
Presence of Lined perennial wells and yellow-wash.
Ans.2 (a) (i) Dadarla- 1020. 
(ii) Bhamra- 0420. 
(b) Dantrai is the largest settlement. PO (Post office) is one communication facility.
(c) R.F. stands for Representative Fraction. It is the ratio between the distances on the map to the
distance on the corresponding ground and is in fraction. The numerator denotes the length on the
map and the denominator denotes the actual distance on the ground. E.g. 1:50,000
Distance on the map (MD
R.F. =
Distance on the ground (GD)
R.F. of the map is 1:50,000 i.e. 1 unit on the map is equal to 50,000 units on the ground.
(d) Total grids from Eastings 02-10 (West to East) is 9
Total grids from Northings 17-28 (South to North) is 11
Each side of grid is 2 cm,
Scale- 2 cm = 1 km
From Eastings 02-10 (West to East) – 18 cm (9×2) = 18 ÷ 2= 9 km (L)
From Northings 17-28 (South to North)- 22 cm (11 × 2) = 22 ÷ 2 = 11 km (B)
Area = L×B
Therefore, 9 x 11 = 99 sq.km.
(e) Contour Interval- is the vertical difference between two consecutive contour lines which remains
constant throughout the map, e.g. 20 metres.
Contour Interval of the map is 20 meters.
(f) Highest spot height is 339 and the lowest spot height is 263. They are in metres.

P-8 G E OG R A P H Y - X
(g) (i) Spot height 339- 042276.
(ii) Permanent hut located near Dhad Talao- 077225.
(h) (i) Main occupation – Agriculture.
(ii) Religion- Hindu.
(i) Natural source of irrigation- Sukli Nadi and seasonal streams.
Man made source of irrigation- Lined perennial well.
(j) Mode of transportation- Cart track. Special feature of this mode is Motorable in dry season.
ll

S O L U T I ON S P-9
DRAMA
CHAPTER

LOCATION, EXTENT AND PHYSICAL FEATURES


12
WORKSHEET-9
Ans.1

(b) Chenab (e) 82½ ˚E


(i)
Bihar
(Highest density
of population)
(g)
Nathu-la Pass

(j) Allahabad
(h) Jharia (Coal)
(h) Singhbhum (Iron)

(d) Red
Soil
(a) Eastern
Ghats

(c)
SW Monsoon
Winds of
(a) Western Bay of Bengal
Ghats Branch
(f) Coromandel
Coast

Map not to scale


S O L U T I ON S P-11
Ans.2

(h)
Delhi
(c) Garo Hills

(a) Tropic of C (d) C


ancer

(i)
(f) Black
Gulf of Soil
Kutch (b) (e)
Godavari NE Monsoon Winds

(g)
(j) Coromandel Coast
Kerala receives rainfall in
(Densely Oct-Nov
populated
state in
South India) Map not to scale

P-12 G E OG R A P H Y - X
Ans.3  

(a) 82½˚E

(f)
Western Rajasthan
Sparsely
populated

(e)
(h)
Vindhayas
Kolkata

(b)
(j) Mahanadi (c)
Mumbai Chilka Lake
High

(i)
(g) SW Monsoon
Alluvial Winds
Soil

(d) Gulf of Mannar Map not to scale



[ICSE Marking Scheme, 2016]

ll

S O L U T I ON S P-13
WORKSHEET-10
Ans.1
(j) Karakoram Pass

(g)
Western
Disturbances
or Westerly Winds

(d) Gomti

(h) Digboi
(b) Malwa
(f) Allahabad

(i)
Laterite
Soil
(a)
Nilgiris

(e)
(c)
Andaman
Malabar
Sea
Coast
Map not to scale

[ICSE Marking Scheme, 2015]

P-14 G E OG R A P H Y - X
Ans.2
(a) 82½˚E (Standard Meridian)

(e)
Karakoram (f)
Range Arunachal Pradesh
Sparsely Populated

(d)
Nathu-la Pass
(b) Yamuna

(j)
(S)

(g)
(c) Laterite Soil (i)
Gulf of SW Monsoon
Khambhat Winds

(h) Vishakhapatnam

Map not to scale


[ICSE Marking Scheme, 2014]

S O L U T I ON S P-15
Ans.3

(e) Indo-Gangetic Plains

(c) Chota Nagpur Plateau

(i) Jharia
(h) Satpura
(d) (f)
Gulf of Red Soil
Kutch
(j) Northern Circars
(b)
Godavari

(g)
South West (a) Chennai
Monsoon Winds

Map not to scale


ll

P-16 G E OG R A P H Y - X
WORKSHEET-11
Ans.1
(e) Karakoram Pass

(j)
(h) Srinagar
(O)

(d) Aravalli

(a) Tropic of C
ancer 23½ ˚N

(b) Narmada

(g) (c) Chilka Lake


Black
Soil

(i)
(f) N.E. Trade Winds
Kerala
(Densely
Populated)
Map not to scale

S O L U T I ON S P-17
Ans.2

(j)
Western Uttar Pradesh
Densely Populated Area

(c)
Malwa
Palteau

(i) Jharia

(h)
Western Ghats
(d)
Konkan
Coast (b) Krishna

(f)
(g)
Alluvial Soil
SW Monsoon
Winds

(a) Kochi

(e) Gulf of Mannar Map not to scale

P-18 G E OG R A P H Y - X
Ans.3

(i) Assam
(a) Chenab (Tea growing region)

(b) Chambal

(g) Sunderbans

(f) Hyderabad
(j) (d) Eastern Ghats
SW Monsoon
Winds

(h) Palk Strait


(c) Nilgiri Hills

Map not to scale


S O L U T I ON S P-19
Ans.4

(a) Satluj

(g)
Laterite
(c) Aravalli Soil

(i) Tropic of C
ancer

(h) (b) Godavari


Offshore
oilfield
(j) Bay of
Bengal
current

(f) Bangaluru

Map not to scale


ll

P-20 G E OG R A P H Y - X
WORKSHEET-12
Ans.1

(d) Ravi

(e) Khasi

(a) Satpura
(h) Laterite Soil

(b) Krishna

(f)
N.E. Monsoon Winds

(g) Malabar Coast


Map not to scale

S O L U T I ON S P-21
Ans.2

(c) Indus

(b) 82½˚E Longitude

(f) Brahmaputra

(d) (a)
Kathiawar Mt. Everest
Peninsula

(e)
Gulf of
Map not to scale
Mannar

P-22 G E OG R A P H Y - X
Ans.3

d (ii) Yamuna

c (ii) Thar Desert

c (i)
Red Soil

a (ii)
Western
Ghats c (iii)
Region of
Winter Rainfall

Map not to scale


S O L U T I ON S P-23
Ans.4
a (i) Karakoram Range

d (i) 1. Arunachal Pradesh


sparsely populated

b (ii)
Black
Soil

T a (ii) Eastern Ghats

c (iii) Godavari
d (ii) Direction of Summer Monsoon

a (iii) Nilgiris

Map not to scale



ll

P-24 G E OG R A P H Y - X
DRAMA
CHAPTER

CLIMATE
13
 WORKSHEET-1
Ans.1 Retreating North –East Monsoon winds.
Ans.2 (i) Since Agra is situated in the interior part of the country and is experiencing the Continental type
of climate and Mumbai is a coastal city experiencing equable climate therefore annual range of
temperature at Agra is more.
(ii) North West part of India like Punjab, Delhi, Uttarakhand.
Ans.3 (i) The Aravalli Hills lie parallel to the South West Monsoon winds coming from the Arabian Sea and thus
cannot shed any moisture in Western Rajasthan.
(ii) The temperature of the region is so high in summers that the monsoon winds get dry and do not shed
any rain there.
Ans.4 (i) Tropical monsoon climate.
(ii) Effect of monsoon wind / Latitudinal extent / Tropic of cancer passing through its centre / effect of
Himalayas. [ICSE Marking Scheme, 2016]
Ans.5 (i) South West monsoon winds from the Bay of Bengal branch.
(ii) Mango Showers or Cherry Blossoms. [ICSE Marking Scheme, 2016]
Ans.6 Rainfall in Rainfall in
June – Sep Dec – Feb
Orographic Cyclonic
Heavy/torrential Light
Causes destruction Beneficial
[ICSE Marking Scheme, 2015] [Any two]
Ans.7 (i) Station A
Annual Range of temperature= Maximum temperature – Minimum temperature
32.5 – 24.5= 8°C
(ii) 183.2 cms.
(iii) Station A is situated on the eastern coast. Retreating North-East winds bring rainfall to this region.

Ans.8 (i) Kanpur lies in the interior, far from moderating effect of the sea.
(ii) Kochi is warmer because it is closer to the Equator than Mumbai.
(iii) It is because the Monsoon winds come to India from the south west and so reach the west coast before
it can reach the Ganga plain. [ICSE Marking Scheme, 2016]

Ans.9 (i) Annual rainfall- 128.7 cm.


(ii) 32.5° C – 24.5° C = 8°C
(iii) East Coast. It gets most of the rain in the season of retreating monsoon.
[ICSE Marking Scheme, 2016]
Ans.10 (i) Hottest month- May.
(ii) Annual rainfall- 131.9 cm.
(iii) Retreating Monsoon.
Ans.11 (i) 36.2° C – 24.6° C= 11.6° C
(ii) Annual Rainfall- 140.9 cms.
(iii) The station is located on the east coast because it gets most of its rain in the winter months.
Ans.12 (i) Annual Rainfall- 114.3 cms.
(ii) Chennai, because it is receiving most of its rainfall from October to December.
(iii) Retreating Monsoon Season.
Ans.13 (i) Mean Annual temperature- 28°C

S O L U T I ON S P-25
(ii) Total rainfall- 180.5 cms.
(iii) The station has a maritime climate as the range of temperature is very low i.e. 6.6°C.
Ans.14 (i) It is due to the effect of land and sea breeze caused by the differential rate of heating and cooling of
the land and sea which results in an equable and moderate climatic conditions.
(ii) The Aravalli hills lie parallel to the south west monsoon winds of the Arabian Sea branch and do
not hinder the winds. Thus, Rajasthan remains dry. On the other hand, the winds blowing from the
Bay of Bengal branch sheds its moisture on its way and gets dry by the time it reaches Rajasthan.
Rajasthan also lies on the leeward side of the Aravalli Hills and thus receives less than 25 cm of rainfall.
(iii) The Coromandel Coast, situated on the eastern coast, receives rain in winter due to the cyclonic
depressions that prevail over the Bay of Bengal due to low pressure conditions. It doesn’t receive rain
in summer as it lies in the leeward side of the Arabian Sea branch.
ll
 WORKSHEET-14
Ans.1 Jaipur is situated in the interior part of the country, far away from the influence of the sea while
Mumbai is a coastal city and is influenced by sea. Thus, Jaipur has a higher annual range of temperature
than Mumbai. 
Ans.2 (i) Kerala- Mango showers are good for the tea and coffee crop.
(ii) West Bengal- Nor’wester is good for the rice and jute crop.
[ICSE Marking Scheme, 2014]

Ans.3 Punjab - rain from the westerly depressions/western disturbances. Tamil Nadu - rain from the north
east monsoons. [ICSE Marking Scheme, 2014]
Ans.4 Mountains or relief is responsible for the distribution of rainfall in India. For example, the Western
Ghats act as a barrier for the south west monsoon winds which results in heavy rainfall on the
windward side of the Ghats while the leeward side of the Western Ghats remains dry.
Ans.5 Mumbai lies on the windward side of the Western Ghats and receives rainfall from the south-west
monsoon in the summer season while Chennai is located on the eastern side and receives rainfall from
the retreating north-east monsoon winds which picks up moisture from the Bay of Bengal and shed
rainfall there.
Ans.6 (i) The driest month is March with the least rainfall of 1.3 cms.
(ii) The annual rainfall is 129.2 cms.
(iii) The annual range of temperature is 32.5 – 20.4 = 12.1°C
Ans.7 (i) The North East monsoon winds blow over most part of India from land to the sea and thus, don’t
bring any rain to most of India.
(ii) The Mango Showers are beneficial for the growth of mango, tea and coffee.
(iii) The latitudinal extent of India is between 8° N to 37° N and the Tropic of Cancer passes through the
centre of India. The southern part of India lies in the tropical zone and experiences high temperature
throughout the year while the northern part lies in the temperate zone and experiences extremity of
temperatures i.e. too cold in winters and too hot in summers.
Ans.8 (i) Annual Range of temperature of Station B is 30.0°C – 24.4°C= 5.6°C.
(ii) Annual rainfall of Station A is 67.4 cms.
(iii) Station B since it receives heavy amount of rainfall in the month of June- July, i.e. 50.2 cms and 61.0
cms respectively.
Ans.9 (i) The annual rainfall experienced by the station is 129.2 cms.
(ii) Annual range of temperature is –32.5°C – 20.4°C = 12.1°C
(iii) November is the wettest month.
Ans.10 (i) Annual range of temperature of Station A is 30.0°C – 24.4°C = 5.6°C. The range is a small one because
it is a coastal city.
(ii) Station B has the lower temperature because it is situated at a high altitude of about 800 m above the
sea level.
(iii) Annual rainfall of Station B is 31.8 cms.
ll

P-26 G E OG R A P H Y - X
 WORKSHEET-15
Ans.1 Western coastal plains receive more rainfall because the Western Ghats check the south-west moisture
laden winds to cause heavy rainfall.
Ans.2 The retreating North-East winds which picks up moisture from the Bay of Bengal and shed rainfall in
Tamil Nadu. This rain is beneficial for growing rice and coffee.
Ans.3 The Northern Plains of India experiences continental type of climate. Its main characteristic is that it
is too hot in summers and too cold in winters and has a high range of temperature. 
Ans.4 Mangalore lies on the windward side of the Western Ghats on the west coast of India while Mysore
lies on the leeward side of the Eastern Ghats. During summer, the south west monsoon winds strikes
the Western Ghats and shed good amount of rainfall whereas the east coast receives rain from the
north-east monsoon winds that picks up moisture from the Bay of Bengal. 
Ans.5 The North West part of India, i.e. Punjab, Haryana, Delhi, Uttarakhand, western Uttar Pradesh etc.
receives rain from Western Disturbances. This rainfall is beneficial for the cultivation of Rabi crops i.e.
wheat and barley. 
Ans.6 (i) Annual range of temperature is 33.5°C – 13.7°C = 19.8°C
(ii) Total rainfall is 67.4 cms.
(iii) November is the driest month.
Ans.7 (i) Annual rainfall of Station is 67.0 cms.
(ii) Annual range of temperature in Station B is 33.0°C – 24.4°C = 5.6°C
(iii) Station B has an equable climate.
Ans.8 (i) Annual rainfall of Station A is 68.4 cms.
(ii) Annual Range of Temperature of Station B is 33.1°C -23.1°C = 10°C
(iii) The North East monsoon winds bring most of the rainfall to Station B. From the month of October
to December, the winds while crossing the Bay of Bengal picks up moisture from there, enters the
eastern coasts and strikes the Eastern Ghats and sheds rainfall.
Ans.9 (i) Average Annual Rainfall of Station B is 128.7 ÷ 12 = 10.72 cms.
(Total Annual Rainfall ÷Total no.of months)
(ii) Station A is located away from the sea because the range of temperature is high.
(iii) Annual Range of Temperature for Station A is 33.9°C – 14.9°C = 19°C.
(iv) December is the driest month in Station A.
Ans.10 (i) Annual Rainfall for Station A is 67 cms.
(ii) Annual Range of temperature at Station B is 30.0°C – 24.4°C = 5.6°C
(iii) Station A lies in the Northern Hemisphere.
(iv) Station B has an equable climate.
ll
 WORKSHEET-16
Ans.1 The factors that influence the climate of India are-
(a) Latitude : the further one moves away from the equator, the colder it gets as the rays of the sun get
more oblique. Hence places in the south like Kochi, Chennai, Bangalore and others are warmer than
places in the north like Delhi, Agra, Srinagar and others due to proximity to the equator.
(b) Altitude : the higher one goes above sea-level, the colder it gets as the atmosphere is heated from
below. Hence places at a higher altitude like Simla, Mussoorie, Panchgani and others are cooler than
places at sea level like Mumbai, Bhopal, Jaipur and others.
(c) Distance from the sea : The further one moves away from the sea, the higher will be the range of
temperature. Hence places like Mumbai,
Chennai, Kolkata and others will have an equable climate as they experience the moderating influence
of the sea, while places further inland like Pune, Indore, Hyderabad, Amritsar, Delhi and others will
have a continental type /extreme type of climate.
[Any relevant examples]

S O L U T I ON S P-27
(d) Varied Relief- When mountains lie perpendicular to onshore winds, heavy rainfall occurs on the
windward side and scanty rainfall occurs on the leeward side; and when mountains lie parallel to
rain-bearing winds, little rain falls. E.g Aravallis – Thar / Rajasthan./ So places that lie on the windward
side of mountains like Mumbai, Cherrapunji and others will receive more rain than places that lie
on the leeward side of mountains like Pune, Shillong and others. Alignment of mountain ranges/
Presence of Himalayas- Checking the South West Monsoons from blowing into
Central Asia / Checking Cold winds from Central Asia. Impact of prevailing winds / Upper air circulation /
Jet Stream. [ICSE Marking Scheme, 2015] [Any 2 points]
Ans.2 The direction is South West.
The intense heat during summers in India develops low pressure over the Northern Plains and
high pressure over the sea. This low pressure attracts the moisture bearing winds from the southern
hemisphere. After crossing the equator, they are deflected to the right and blow over India as the
south west monsoon winds.
Ans.3 The cyclonic winds originating over the Mediterranean Sea are called Western Disturbances. These
winds blow towards India in the month of November and December and bring heavy rainfall over the
North West part of India. 
Ans.4 (i) Kerala lies to the SW and hence it is the first state to receive rain from SW Monsoon of the Arabian Sea
branch as these winds move from SW to NE. These winds withdraw from North and move towards
south and hence Kerala is the last state from which these winds withdraw. Thus, Kerala is the first
state to receive the South-West Monsoon and the last to see it withdraw.
(ii) The Konkan coast lies on the windward side of the Western Ghats and hence receives heavy rain from
the SW Monsoon winds from the Arabian Sea branch. The windward side of an area is the slope of a
range that receives heavy rainfall.
(iii) Kanpur lies to the interior, far away from the influence of the sea, whereas Chennai lies along the
coast and is influenced by the sea. Thus, Kanpur is deprived of the moderating influence of land and
sea breeze. [ICSE Marking Scheme, 2015]
Ans.5 (i) Patna gets a much heavier rain than Delhi does because Patna lies in the lower Ganga valley while
Delhi lies in the upper Ganga valley and the Bay of Bengal branch of the South West
Ganga valley. The South west monsoon Bay of Bengal branch deflects when it comes in contact with
the Garo Khasi and Jaintia Hills in Assam moving up the Ganga Valley. The rain shed while following
the line of the.
(ii) Western Rajasthan gets no rain from the Arabian Sea Branch of the South West Monsoon winds
because it blows parallel to the Aravalli mountains and therefore shed no moisture.
(iii) It never gets too cold in Mangalore, not even in December because of the moderating influence of the
land and sea breezes and because it is quite close to the equator. [ICSE Marking Scheme, 2014]
ll
 WORKSHEET-17
Ans.1 (i) India lies in the tropical belt.
(ii) The climate is influenced by the monsoon winds which are largely confined to the Tropics, i.e. between
20°N and 20°S.
Ans.2 Two main features of the Indian monsoon-
(i) Monsoon is erratic and sporadic.
(ii) Monsoon is unevenly distributed.
Ans.3 Tropical Cyclones Temperate Cyclones
1.  These cyclones are thermal in origin. 1.  These cyclones are frontal in origin.
2. The wind velocity of these type of cyclones are 2. The wind velocity in this type of cyclones
very high, i.e., 300km/hr. are very low, i.e. 35 to 70 km/hr.
3.  These cyclones are active in summer season. 3.  These cyclones are active in winter season.
[Any Two]
Ans.4 Due to location, vast extent of the country, presence of the lofty Himalayan Mountain, large coastline
along the eastern and western sides and the Western Disturbances, there is a great variation in the
climate of the Indian Sub-Continent. 

P-28 G E OG R A P H Y - X
Ans.5 (i) Punjab and Uttrakhand.
(ii) The Western disturbances or cyclonic depressions originating over the Mediterranean Sea.
Ans.6 (i) South West Monsoon winds are responsible for the rainfall experienced over the greater part of the
Indian Sub-Continent.
(ii) From June to September.
Ans.7 (i) Shillong lies on the rain shadow area or the leeward side of the Khasi Hills while Cherrapunji lies on
the windward side.
(ii) The moisture laden winds coming from the Bay of Bengal branch gets entrapped in the Garo-Khasi
Hills and causes heavy rainfall.
Ans.8 (i) During the Cold weather or winter season the North East Trade Winds dominates the Indian Sub-
Continent.
(ii) The North East Trade Winds get completely reversed in the Summer Season.
The four months covered by this season are October, November, December and January.
Ans.9 (i) It is so because Tamil Nadu lies in the rain shadow region of Arabian Sea branch and the Bay of Bengal
is parallel to the coast.
(ii) It is so because the northern Plains are located far away from the influence of the sea.
(iii) Central Maharashtra receives little rainfall because it lies in the rain shadow region of the Western
Ghats.
Ans.10 (i) Shimla is situated at a higher altitude than Delhi. Thus, due to lapse rate, as one moves up the altitude,
the temperature decreases.
(ii) The cold winds from the north are prevented from entering into India due to the presence of the lofty
Himalayas and thus helps to keep the temperature of the northern plains at a moderate level.
(iii) Kochi is a coastal city and is influenced by the sea while Agra is situated in the interior part of the
country experiencing continental type of climate. Thus, Kochi has a lesser annual range of temperature
than Agra.
Ans.11 (i) The South West monsoon winds of the Bay of Bengal branch while advancing towards North West
sheds heavy rainfall on Patna as it falls first on its way and then the amount of rainfall decreases by
the time it reaches Varanasi.
(ii) The Aravalli Hills lie parallel to the South West Monsoon winds coming from the Arabian Sea and thus
cannot shed any moisture in Western Rajasthan.
(iii) India has varied climatic conditions because of large latitudinal and longitudinal extent, altitude and
distance from sea.
Ans.12 (i) The mountain slopes are cooler than the plains during summer due to normal lapse rate i.e. with every
165 metres of ascent there is a fall of 1°C of temperature.
(ii) Mumbai is warmer than Kanpur in December because Mumbai is a coastal city and has the influence
of sea which results in moderate climate.
(iii) Rajasthan receives very little rainfall because Aravalli Hills lie parallel to the South West Monsoon
winds coming from the Arabian Sea branch. 
ll

S O L U T I ON S P-29
DRAMA
CHAPTER

SOIL RESOURCES
14
 WORKSHEET-18
Ans.1 (i) Black soil is formed by the weathering of igneous rock.
(ii) Cotton.
Ans.2 (i) Alluvial soil.
(ii) Alluvial soil is formed by the deposition of sediments and silts brought down and deposited by the
rivers.
Ans.3
Ansa

Alluvial Soil Black Cotton soil


1. These soils are formed by the deposition of 1. These soils are formed by the weathering of
silts and sediments brought down by the lava flow rocks or igneous rocks.
rivers.
2. The soil is very fertile and rich in mineral 2. The soil becomes sticky and needs to be tilled
nutrients like potash and lime. immediately after the first rain..

Ans.4 Both are red in colour because of the presence of iron oxide / both are infertile soils / both are friable /
both are not moisture retentive /both are poor in organic matter. [Any two]
[ICSE Marking Scheme, 2016] 
Ans.5 (i) Black soil- moisture-retentive/self-ploughing/rich in lime, iron, calcium, alumina, potash/ deep and
fine grained / clayey … [Any one point]
(ii) Red soil- rich in iron/porous/friable/does not get water-logged/responds to manures or fertilizers /rich
in potash … [Any one point]
[ICSE Marking Scheme, 2015] 
Ans.6
Alluvial Red
Silt and sediments brought down by rivers - Formed by weathering of metamorphic rocks - in
transported soil situ soil
Colour ranges from yellow to brown Red because of iron oxides
Loamy Sandy and friable
Fertile Infertile, responds to fertilizers
Rich in potash and lime Contains soluble salts
Found in northern plains and coasts Along the Found in South India in the interior Parts of
river banks Bihar, Jharkhand, NE states/Orissa/West Bengal.
Ans.7 Formed in-situ as a result of leaching under typical monsoonal conditions with high temperature and
heavy rainfall with alternating wet and dry spells.
Ans.8
Bhangar Khadar
1. Older Alluvium soil. 1. Newer Alluvium soil.
2. It is found above the flood levels of rivers 2. It lies below the flood levels.
and is of a terrace like structure.
3. It is composed of lime nodules or kankar and 3. It is light in colour and is composed of newer
has a clayey composition. deposits.
4. It is not very fertile. 4. It is fertile and is formed of fine silt and clay.

S O L U T I ON S P-31
Ans.9
Alluvial soil of the Northern Plains Alluvial soil on the Coastal Plains
1. This soil has been deposited by the three 1. This soil is formed by the rivers Godavari,
important river systems-Indus, Ganga and Krishna, Narmada and Tapti.
Brahmaputra.
2. These soils are porous, coarse in the upper 2. These soils are non-porous, fine, contains
region, contains minerals especially potash minerals like iron and calcium and are black
and light in colour. in colour.
Ans.10 (i) Maharashtra
(ii) Gujarat
Regur soil help agriculture due to the following qualities :
(i) Regur soil has the quality of self-ploughing.
(ii) Due to the occurrence of deep cracks during period it helps in oxygenation.
(iii) It is moisture retentive. [Any one point]
Ans.11 (i) Laterite soil.
(ii) Laterite soils are formed in-situ as a result of leaching under typical monsoonal conditions with high
temperature and heavy rainfall with alternating wet and dry spells.
(iii) This soil is not suitable for cultivation because it is highly acidic in nature.
Ans.12 (i) Alluvial soil
(ii) Laterite soil
(iii) Alluvial soil
Ans.13 (i) Alluvial soil is coarse in the upper valley of the rivers because the eroded matter is carried away by
the fast flowing river but in the lower course, the river reduces its speed and the soil particles become
finer due to attrition or because the load itself gets eroded.
(ii) Black soil does not get leached because it is clayey and sticky and moisture retentive and therefore the
rain cannot wash out the silicates.
(iii) Khadar is the newer alluvium which keeps getting replenished by the river bringing down more
eroded material. [ICSE Marking Scheme, 2016]
Ans.14 (i) Pedogenesis- the process of soil formation.
(ii) Humus- the decayed organic matter that helps make soil fertile.
(iii) Bhangar- the older,less fertile alluvial soil. [ICSE Marking Scheme, 2015]
Ans.15 (i) Laterite soil
(ii) Alluvial soil
(iii) Laterite soil
Ans.16
Transported Soil In Situ Soil
1. These soils are formed by the deposition of 1. It is formed in their original position by the
silts and sediments brought down by the breaking up of parent rocks, e.g. Black Soil,
rivers, e.g. Ganga, Yamuna, Brahmaputra, etc. Red soil, Laterite Soil, Desert Soil etc.
2. The soil is very fertile and rich in mineral 2. It is less fertile and is deficient in
nutrients like potash and lime. phosphorous, lime, nitrogen and humus.
3. It is found in river valleys. 3. It is found in the Deccan Trap.
Ans.17 (i) Karnataka and Tamil Nadu
(ii) 1. Wheat and rice are well grown on this soil.
2. Responds to manures or fertilizers.
3. Rich in iron and potash. [Any two points]
(iii) Wheat and Rice.
Ans.18 (i) It is fine textured and clayey in nature.
(ii) Its moisture retentive and becomes sticky when wet.
(iii) Rich in lime, magnesium and iron.

P-32 G E OG R A P H Y - X
Ans.19 (i) Laterite soil is not suitable for cultivation because it is highly acidic in nature and is not moisture
retentive.
(ii) Red soils are red in colour due to high content of iron oxide present in it.
(iii) Khadar soils are preferred to Bhangar soils because Khadar soils are fertile due to fine silts and clay as
they are replenished every year by floods. 
Ans.20

Alluvial soils in the Lower Courses Alluvial soils in the Upper Courses
1. Soil is fine. 1. Soil is coarse.
2. It is moist. 2. It is dry.
3. It is rich in humus. 3. It is sandy and porous.
ll
 WORKSHEET-19
Ans.1 (i) Laterite soil is highly acidic in nature.
(ii) It is a leached soil and thus, infertile.
(iii) Rich in iron and poor in silica, lime, nitrogen and humus.
(iv) It is porous and coarse in texture. [Any two points] 
Ans.2 (i) Laterite soils.
(ii) Alluvial soils.
(iii) Black soil, Regur soil or Black cotton soil.
(iv) Black soil.
Ans.3 Laterite soil.
(i) Kerala
(ii) Odisha 
Ans.4 (i) It is porous, friable and coarse.
(ii) It does not retain moisture.
(iii) Rich in iron and potash.
(iv) Deficient in lime, nitrogen, phosphorous and humus.
(v) Responds to manures or fertilizers.
(vi) It is red in colour due to presence of iron oxides. [Any two points] 
Ans.5 Laterite soils are formed in-situ as a result of leaching under typical monsoonal conditions with high
temperature and heavy rainfall with alternating wet and dry spells.
This soil is not suitable for cultivation because it is highly acidic in nature and is not moisture retentive.
Ans.6 (i) Red soil is red in colour due to high iron oxide content.
(ii) Large tracts in Maharashtra are covered with Black soil because it is formed from weathered lava rocks
which have solidified during volcanic activity.
Ans.7 Maharashtra. Two crops grown in this soil are Cotton and Sugarcane. 
Ans.8 With the help of growing leguminous crops after harvesting of cereal crops and from the fertilizers,
soil can get nitrogen.
Ans.9 This soil is not suitable for cultivation because it is highly acidic in nature and is not moisture retentive.
This soil is found in the Summits of Eastern Ghats and Western Ghats, West Bengal, Odisha. 
[Any one place]
Ans.10 It is a process by which soluble substances from the soil are removed through percolation.
A region, south of the Tropic of Cancer, formed by ‘Leaching’ is the Highland areas of peninsular
Plateau.
Ans.11 (i) Black soil is largely found in the Deccan Trap region because it is formed by the denudation of volcanic
rocks, found in that region.
(ii) Khadar is composed of newer alluvium deposits and is fertile as it is formed of fine silt and clay.
(iii) Soil erosion by wind is common in arid regions because these regions are devoid of vegetation and
moisture due to scanty rainfall and as such the soil is blown away by strong winds. 
Ans.12 (i) Transported soil : These soils are formed by the deposition of silts and sediments brought down by
the rivers, e.g. Ganga, Yamuna, Brahmaputra, etc.

S O L U T I ON S P-33
(ii) In situ : It is formed in their original position by the breaking up of parent rocks, e.g. Black Soil, Red
soil, Laterite Soil, Desert Soil etc.
(iii) Humus : It is the organic matter present in the soil formed by the decomposition of plants and animals
that helps to make soil fertile.
Ans.13 Two characteristics of Black soil :
(i) It is fine textured and clayey in nature.
(ii) It has a self-ploughing quality.
(iii) Its moisture retentive and becomes sticky when wet and forms crack when dry.
(iv) Rich in lime, magnesium, potash, aluminium and iron.
(v) Poor in phosphorous, nitrogen and organic matter.  [Any two points]
This soil is agriculturally important because it is moisture retentive and is rich in minerals like lime,
magnesium, potash, iron and aluminium.
Ans.14 These soils are formed by the deposition of silts and sediments brought down by the rivers.
This soil is agriculturally important because it is rich in potash, lime and humus of the Ganga deltaic region.
Ans.15 Red soil is formed by the weathering of old hard crystalline and metamorphic rocks. The reasons for
its low productivity is due to deficiency of lime, nitrogen, phosphorous and humus in the soil.
Ans.16 Regur soil is formed by the weathering of lava flow rocks or igneous rocks.
Four important properties of Regur soil are :
(i) It is fine textured and clayey in nature.
(ii) It’s colour vary from deep black to chestnut brown.
(iii) Its moisture retentive and becomes sticky when wet and forms crack when dry.
(iv) Rich in lime, magnesium, aluminium and iron. 
ll
 WORKSHEET-20
Ans.1 (i) Sheet erosion- The foothills of Shiwalik range.
(ii) Gully erosion- Chambal Valley region. 
Ans.2 Removal or destruction of top soil is called soil erosion.
Terrace farming / contour ploughing / crop rotation etc. [ICSE Marking Scheme, 2016] 
Ans.3 (i) Leaching
(ii) Gully Erosion [ICSE Marking Scheme, 2015] 
Ans.4 Any two of the following:
Terrace farming / contour ploughing Strip cropping
Preventing over grazing Planting shelter belts and trees
Constructing dams and plugging gullies
Introducing better methods of cultivation, use of fertilizers, rotation of crops
 [ICSE Marking Scheme, 2014] 
Ans.5 (i) Contour Ploughing : Contours act like bunds. Ploughing along contours on a slope prevents soil being
washed away by rainwater or by surface run-off.
(ii) Constructing Dams : Rivers cause soil erosion, therefore, dams are built in the upper course to control
the erosion. 
Ans.6 Soil Conservation is the effort made by man to prevent soil loss from erosion or reduced fertility
caused by over usage.
Re-afforestation is planting of trees in lieu of the number of trees being cut. It helps as the roots of the
trees planted again hold the soil tightly and prevents it from eroding. 
Ans.7 (i)
Sheet Erosion Wind Erosion
It occurs on gentle slopes where the thin layer It takes place where there is less or no vegetation
of top soil is removed due to rain water. due to high velocity an strong movement of
winds and deposition of soil particles.
(ii) Man is largely responsible for soil erosion because he is involved in deforestation, overgrazing of
domestic animals, shifting agriculture and faulty farming practices.

P-34 G E OG R A P H Y - X
Ans.8 (i) The removal or destruction of the top soil is called soil erosion.
(ii) Soil erosion is caused due to increasing population pressure on land, overgrazing by domestic animals,
floods, deforestation and bad farming techniques.
(iii) (a) Scheme of Integrated Watershed Management.
(b) Scheme for Reclamation and development of ravine areas.
(c) Scheme for Control of Shifting Cultivation.
(d) National Project on Development and use of Bio-Fertilizers and National Project on Quality
Control implemented.
(e) Rainwater Harvesting.
Ans.9 (i) Sheet erosion is the slow removal of a thin layer of soil by rainwater washing it away.
(ii) Soil conservation refers to the efforts made to prevent soil from getting eroded.
(iii) In situ means to develop in one area without any movement. It refers to residual soil.
[ICSE Marking Scheme, 2016]
Ans.10 (i) As it contains minerals like iron, potash, lime../is loamy/has sufficient depth/is renewed annually/ It is
a transported soil which brings along lots of minerals … [Any one point]
(ii) As soil supports all plant life/to increase our agricultural output/ Various methods/ Efforts made by
man to check soil erosion and retain the fertility of the soil… [Any one point]
(iii) As it helps prevent soil erosion/holds the soil together/helps maintain the ecological balance/checks
global warming/reduces severity of drought… [Any one point]
[ICSE Marking Scheme, 2015]
Ans.11 (i) Different regions in India have different kinds of soil because of the different rocks which get
weathered to form soil and the different agents of erosion minerals present / difference in temperature
and rainfall.
(ii) Black soil is suitable for the growth of cotton because it is moisture retentive, has self- ploughing
qualities and is rich in lime, iron, potash, calcium, claying in nature.
(iii) Soil is a natural resource which must be conserved or else the land becomes barren and cannot be
cultivated, food crops will have to be imported and the agro-based industries will suffer. Some kinds
of soil are also required for construction purposes. It takes very long for an inch of top soil to be
formed. [ICSE Marking Scheme, 2014]
Ans.12 Two important agents of erosion are- Running Water and Wind.
(i) Running water- It can be controlled by methods like contour ploughing, terracing, plugging gullies
and planting trees.
(ii) Wind- It can be controlled by the methods like strip cropping to check the impact of the winds, by
planting trees along the edges of the fields, waste lands and on steep slopes.
Ans.13 Soil Conservation is the preventing of soil loss from erosion or reduced fertility caused by over usage.
Two farming techniques which help in soil conservation are-
(i) Contour Ploughing
(ii) Terrace Farming.
Ans.14 India is an agricultural country and its economy depends on it. So soil conservation is needed to check
floods, soil erosion, etc.
Two methods of soil conservation are-
(i) Contour ploughing
(ii) Planting trees or Afforestation.
Ans.15
Sheet Erosion Gully Erosion
1. It is the slow removal of a thin layer of soil 1. Due to heavy rainfall deep gullies are formed
when vegetation is destroyed. on the soil and remove the soil nutrients.
2. Rainwater washes away the thin layer of 2. It removes nutrients and heavy load of loose
bare soils. soils, making the soil unproductive.
3. It occurs on gentle slopes. 3. It occurs in the Chambal Valley region.
ll

S O L U T I ON S P-35
DRAMA
CHAPTER

NATURAL VEGETATION
15
 WORKSHEET-21
Ans.1 It is Littoral or Mangrove or Tidal Forests. Sundari tree is found there.
Ans.2 Tropical Monsoon Deciduous Forests are the most commercially important vegetation belt in India.
The rainfall range is between 100 cm to 200 cm.
Ans.3 Teak.
It has high oil content making it very weather resistant. It has great value as timber and is very heavy,
tough and durable. It is broadly used for making furniture, houses and also ships. The bark of the teak
tree is considered to be astringent and is used to treat bronchitis.
Ans.4 (i) The trees have long roots which enable them to obtain water from deep underground.
(ii) The leaves are small which minimises evaporation and the stems are succulent which helps to conserve
water.
Ans.5 (i) Teak
(ii) Sundari
Ans.6 (i) Tropical deciduous forest
(ii) Teak / Sal / Shisham / Sandalwood [ICSE Marking Scheme, 2016]
Ans.7 (i) Tropical evergreen forest
(ii) Due to heavy rainfall of above 200 cm [ICSE Marking Scheme, 2016] 
Ans.8 Any two of the following:
(i) Tropical deciduous forests have trees found in pure stands.
(ii) Trees shed their leaves in a particular season Provide valuable timber and other forest products.
 [ICSE Marking Scheme, 2015] 
Ans.9 (i) 1. Due to scanty rainfall, the forests have xerophytic vegetation.
2. The trees of the forests are stunted and have large patches of coarse grasses.
(ii) An example of desert vegetation-Babool
Its uses- It has high medicinal value. It is a source of gum and used as an emulsifier. The twigs and
barks are chewed to prevent Vitamin C deficiency. Its wood is used as fire wood and charcoal and for
boat building.
Ans.10 (i) Tropical Evergreen Forests.
(ii) Montane Forests.
(iii) Tropical Desert Forests.
Ans.11 (i) Mangrove / Littoral forest or Tidal forest
(ii) Tropical Evergreen forest / Tropical rain forest
(iii) Mountain forest.  [ICSE Marking Scheme, 2016] 
Ans.12 (i) Deodar.
(ii) Sundari.
(iii) Mahogany or Rosewood. [ICSE Marking Scheme, 2015] 
Ans.13 (i) Sundri - firewood / boat building / construction.
(ii) Sandalwood - extraction of sandalwood oil which is used in soaps and cosmetics / handicrafts /
(iii) Rosewood – furniture / floor boards / veneer / wagon parts. [ICSE Marking Scheme, 2014] 
ll
 WORKSHEET-22
Ans.1 They are dense and have a variety of trees and shrubs.
Ans.2 Tropical evergreen forests are dense/ Heavy logs make it difficult for accessibility/ Trees are found in
mixed stands/ Dense undergrowth…  [Any two points]
[ICSE Marking Scheme, 2015] 

S O L U T I ON S P-37
Ans.3 Any two of the following :
(i) Provide valuable timber and other forest products.
(ii) These forests are not as dense as the tropical evergreen forests. The wood of the trees is not so heavy
as to be difficult to transport.
(iii) These trees grow in stands and are therefore easier to exploit for commercial purposes.
[ICSE Marking Scheme, 2014] 
Ans.4 The characteristic of Tropical Rain Forests are-
(i) They are dense and have a variety of trees and shrubs.
(ii) Trees reach at a height of 60 m or above.
(iii) Due to thick canopy of trees, herbs and grasses cannot grow.
(iv) These forests do not have any fixed period of time for shedding of leave.  [Any two points] 
Ans.5 (i) They are dense and have a variety of trees and shrubs which makes it difficult to exploit.
(ii) Due to lack of proper transportation facilities.
Ans.6 (i) These forests are dense, evergreen and of varying heights.
(ii) The trees have long roots submerged under water and have pores which help them to breathe during
high tide. 
Ans.7 The forests grown along the sea coasts, in wet marshy areas, in river deltas, in tidal or other swampy
areas., e.g., in the areas around the eastern coasts, in the saline swamps of Sunderbans in West Bengal,
etc.
Sundri and Gorjan are the two typical trees found there.
Ans.8 (i) The delta of the Ganga river - mangrove forests / sundarbans / tidal forests
(ii) The windward side of the Western Ghats - tropical evergreen forests
(iii) The Deccan Plateau - monsoon deciduous forests/ thorn forests [ICSE Marking Scheme, 2014] 
Ans.9 (i) Semul.
(ii) Mahogany.
(iii) Sandalwood. 
Ans.10 (i) 1.  Tidal Forests- Sunderbans in West Bengal or Ganga Delta Region.
2.  Thorn and Scrub Forests- Rajasthan.
(ii) Thorn and scrub forests are found in this region because these forests receive less than 50 cm of
rainfall.
Ans.11 (i) Teak- It has high oil content making it very weather resistant. It has great value as timber and is very
heavy, tough and durable. It is broadly used for making furniture, houses and also ships. The bark of
the teak tree is considered to be astringent and is used to treat bronchitis.
(ii) Sal- It is hard, tough and heavy wood. It is mainly used for making doors, windows, railings of bridges,
beams, railway sleepers.
(iii) Sandalwood- It is widely used in the cosmetic industry. It is used as perfume and its oil is used in
making aromatic substances. It is also used for making ornamental objects like statues.
Ans.12 Tropical Evergreen Forests are found in the western part of Western Ghats.
Two reasons are-
(i) This region receives an annual rainfall of more than 200 cm.
(ii) The trees remain evergreen. 
ll
 WORKSHEET-23
Ans.1 (i) Forests control humidity, lower the temperature and can be responsible for rain.
(ii) The roots of the trees check the speed of running water which then does not overflow its banks and
also because the roots take in some of the water. [ICSE Marking Scheme, 2014] 
Ans.2 (i) Forest plays an important role in modifying climate and weather.
(ii) They also play an important role in controlling soil erosion by wind and water. 
ll

P-38 G E OG R A P H Y - X
 WORKSHEET-24
Ans.1
Agro Forestry Social Forestry
1. It is a sustainable land use system that 1. It is the management and protection of
maintains or increases the total yield by forests and afforestation of barren and
combining food crop together with forest tree deforested lands with the purpose of helping
and livestock ranching on the same unit of in the environmental, social and rural
land. development.
2. It aims to provide soil conservation to improve 2. It aims at raising plantations by the common
the growth of forest products and agricultural man so as to meet the growing demand
crops together. for timber, fuel wood, fodder, etc., thereby
reducing pressure on traditional forest areas.
Ans.2 The reasons are as follows :
(i) Land was cleared for cultivation, industrialisation and urbanisation.
(ii) Shifting agriculture was practised.
(iii) Forest was cut recklessly for raw material. [ICSE Marking Scheme, 2016] 

Ans.3 Afforestation Deforestation


Planting of trees over a large area Cutting of trees over a large area
Disadvantage : Leads to soil erosion/extinction of flora
and fauna/rise in temperatures/floods/increases severity of
drought helps maintain the ecological balance/checks global
warming/reduces severity of drought…  [Any one point]
[ICSE Marking Scheme, 2015] 
Ans.4 (i) To maintain the environmental stability.
(ii) To preserve natural forests through wide variety of flora and fauna.
(iii) To checksoil erosion. 
Ans.5 Three methods for the Conservation and Development of Forests :
(i) To plant trees or Afforestation.
(ii) Banning the practice of shifting cultivation prevalent amongst the tribals.
(iii) To discourage cutting of trees and ensure to plant ten saplings in lieu of felling one tree.
ll

S O L U T I ON S P-39
DRAMA
CHAPTER

WATER RESOURCES
16
 WORKSHEET-25
Ans.1 Tamil Nadu, Karnataka, Rajasthan, Maharashtra, etc. [Any two states]
Ans.2 (i) High demand of water due to increasing population is leading to the lowering of the ground water
levels.
(ii) Rainfall in India is seasonal, erratic and unreliable and thus the farmers cannot wholly depend on
rain.
(iii) More than 90% of water is utilized for irrigation.
(iv) The Industries also utilizes a lot of water and pollutes it too. [Any two points]
Ans.3 Two main reasons why water scarcity occurs in India-
(i) Increasing demand of water for domestic and industrial uses.
(ii) Lack of Watershed Management.
(iii) Rainfall is erratic and unreliable. [Any two]
Ans.4 (i) Rainwater Harvesting
(ii) 1. Harvesting surface and groundwater,
2.  Prevention of losses through evaporation and seepage,
3. Other techniques aimed at conservation and efficient utilization of limited water endowment.
 [Any two points] 
Ans.5 (i) It is a technique of increasing the recharge of ground water by capturing and storing rain water.
(ii) To make water available for future use. [Any two points]
(iii) To avoid flooding of roads. [ICSE Marking Scheme, 2016] 
ll
 WORKSHEET-26
Ans.1 Rainwater harvesting helps to meet the increasing demand for water/ helps to check surface run-off
that chokes the drain / recharge groundwater
Ans.2 Two advantages of rainwater harvesting :
(i) Prevents wastage of water.
(ii) Reduces soil erosion.
(iii) Recharge groundwater resource. [Any two points]
Ans.3 (i) Severe water shortage is due to wastage / pollution/ overuse/ mismanagement / Increasing population /
increasing Industries.  [Any two points]
(ii) make rainwater harvesting mandatory in every housing colony/levy fine on wastage/pass laws to
ensure water is recycled in factories. [Any two points]
[ICSE Marking Scheme, 2015] 

Ans.4 Any three of the following :


(i) The growth of population has resulted in water scarcity.
(ii) Deforestation has led to drought and reduction of rainfall.
(iii) The underground water has been exploited and the water table has been Lowered.
(iv) Much of the water is used up for cultivation Water is required also for industries.
(v) Much of both the ground water and the surface water has got polluted.
[ICSE Marking Scheme, 2014] 
Ans.5 (i) Two methods of water harvesting in India-
1.  Surface runoff harvesting
2.  Roof Top-Water harvesting.

S O L U T I ON S P-41
(ii) 1. Harvesting surface and groundwater.
2.  Prevention of losses through evaporation and seepage.
3.  Techniques aimed at conservation and efficient utilization of limited water endowment.
4.  To make water available for future use.
5.  To avoid flooding of roads and improve ground water quality. [Any two points]
Ans.6 (i) Roof Top Water harvesting
(ii) Surface runoff harvesting
(iii) Khul and Johads in Western Himalayas and in Rajasthan respectively.
ll
 WORKSHEET-27
Ans.1 Perennial canals are more useful than inundation canals because perennial canals contain water
throughout the year and never dry up while inundation canals are seasonal.
Ans.2 Uttar Pradesh
Ans.3 (i) Significance of Irrigation in Punjab-
1.  Availability of water is highly uneven and variable.
2.  It is an agricultural state and along with wheat rice is cultivated at a large scale. (Any one)
(ii) Significance of Irrigation in Rajasthan-
1.  Rajasthan lies in the semi-arid region with less than 25 cm of rainfall.
2.  The groundwater level is available only at a depth of 30 to 60 m. [Any one]
Ans.4 Rain occurs mainly in four months
Some crops like rice and sugarcane need more water.
To cater to increasing demand of food and cash crops there is a need to grow crops all the year round.
 [ICSE Marking Scheme, 2016]

Ans.5 Fertile agricultural land


Cheap electricity
Large amount of underground water. [ICSE Marking Scheme, 2016]
Ans.6 Irrigation is required as rainfall is erratic and unevenly distributed, Irrigation is required to grow
crops throughout the year as rainfall is seasonal, Irrigation is required as some crops like rice and jute
require more water than supplied by rain, Irrigation is required to increase agricultural output by
providing the right amount of water at the right time. Multiplicity of crops / Commercialised crops/
Nature of Soil / Monsoon is seasonal. [Any two points]
[ICSE Marking Scheme, 2015]
Ans.7 Modern methods of irrigation- Drip irrigation, Spray irrigation, Sprinkler irrigation.[Any two types]
Reason- they prevent wastage of water/provides the right quantity of water at the right time / No
waterlogging/ No soil erosion [Any one reason]
[ICSE Marking Scheme, 2015]
Ans.8 Well irrigation is still popular as it is cheap/easy to construct/occupies a small area/control of water is
in the hands of the farmer / Can be dug at any convenient place/ Independent source of Irrigation.
 [Any two points]
[ICSE Marking Scheme, 2016]
Ans.9 (i) (i)   Uncertainty of rainfall.
(ii)   Uneven distribution of rainfall.
(iii)  Requirement of different quantities of water for various crops for their growth.
(iv)  Dependent of crops on the nature of soil.
(v)   Utilization of river water effectively.
(vi)  To increase or maximize production. [Any two points]
(ii) Soft soil and flat level land.

P-42 G E OG R A P H Y - X
Ans.10 (i) (i)   It is inexpensive as they are mostly natural.
(ii)  It is highly beneficial in the uneven rocky plateau of Deccan since rainfall is seasonal.
(iii) It is highly significant because it stores the abundant rainwater and reduces the waste of the
excessive flowing water. [Any one point]
Tank irrigation is commonly used in South India like Andhra Pradesh, Telangana, etc.
(ii) Tubewells are deeper wells from where the water is lifted from a great depth of 20-30 m by the use of
power pumps.
It is reliable during dry season when the surface water dries up since the tube well is drilled upto the
permanent water table.
(iii) It supplies the water to the roots of the plants slowly through pipes, valves, tubing etc. and thus saves
water and fertilizer. This helps in the reduction of evaporation.
Ans.11 (i) It is so because it is water saving as there is no loss of water through seepage or evaporation.
(ii) It is able to irrigate a larger agricultural land and a large amount of underground water is easily
available.
(iii) It is because Northern plains have a good network of perennial rivers which supply water to the
canals throughout the year.
Ans.12 (i) Canal irrigation is popular in Northern plains because it has perennial rivers and land is soft enough
for canal to be constructed from the rivers to the fields.
(ii) Being in Deccan region Karnataka has natural depressions and hard sub surface rocks which make
tank irrigation important.
(iii) As it helps to conserve water.  [ICSE Marking Scheme, 2016] 
ll
 WORKSHEET-28
Ans.1 (i) Uttar Pradesh
(ii) Andhra Pradesh.
Ans.2 Uttar Pradesh and Punjab.
Ans.3 It is less expensive and can be dug anywhere where the soil is soft.
Ans.4 Deccan Plateau is made up of hard impermeable rock which does not allow the rainwater to
percolate underground. The Deccan Plateau is highly dissected and so has many natural hollows and
depressions. [ICSE Marking Scheme, 2014]
Ans.5 Surface wells rely on ground water and not on rivers to flood. They can be dug anywhere on the farm
and the farmer will have water even if he is not too close to a river/cheaper.
[ICSE Marking Scheme, 2014]
Ans.6 Two states- Uttar Pradesh and Punjab
Its importance-
(i) Less expensive and a reliable source of irrigation.
(ii) Can be dug anywhere where the soil is soft. [Any one]
Ans.7 Inundation canals are converted to perennial canals due to the following reasons-
(i) Inundation canals are non-perennial and dependent on rain.
(ii) It cannot supply water on demand or as and when required.
(iii) These type of canals can irrigate only low land areas. [Any two points]
Ans.8 Two disadvantages of tank irrigation are-
(i) In the absence of rainwater during dry season, the tanks become dry and fail to provide water for irrigation.
(ii) Due to deposition of sediments, the tanks get silted up soon and desilting is necessary for making
irrigation suitable which is expensive.
(iii) Tanks occupy large fertile areas which otherwise could be used for agricultural purposes. [Any two]
Ans.9 (i) Large amount of underground water is easily available.
(ii) It is reliable during dry season when the surface water dries up since the tube well is drilled upto the
permanent water table.

S O L U T I ON S P-43
Ans.10 Four disadvantages of tank irrigation in South India-
(i) Deccan Plateau is made up of hard impermeable rock which does not allow the rainwater to percolate
underground.
(ii) The Deccan Plateau is highly dissected and has many natural hollows and depressions.
(iii) In the absence of rainwater during dry season, the tanks become dry and fail to provide water for
irrigation.
(iv) Since tanks are very extensive and shallow, huge quantities of stored water go waste as it gets
evaporated or sinks underground.
Ans.11 Any one geographical reason of the following:
(i) The rainfall is erratic; It is seasonal
It is unevenly distributed over the land
Some crops require more water than the amount provided by the rain.
Some crops need water during the period when there is no rain.
(ii) No loss of water through evaporation. Judicious use of water with no wastage
The flow of water can be customised so as to be beneficial to each crop.
(iii) Percolation of water through the canals can lead to swamps Alkaline salts from the water table
below can reach the surface and make the soil unproductive
Due to water logging the capacity of the soil to absorb water decreases and the water which then
collects can ruin the crop. [ICSE Marking Scheme, 2014] 
Ans.12 (i) Punjab and Haryana
(ii) Two reasons are :
1. There are a number of perennial rivers.
2. In these states large tracts of land are under cultivation which requires water throughout the year
for irrigation.
Ans.13 Two advantages are-
(i) It is able to irrigate a larger agricultural land.
(ii) Large amount of underground water is easily available.
(iii) It is reliable during dry season when the surface water dries up since the tube well is drilled upto the
permanent water table.  [Any two points]
One disadvantage is-
(i) It is costly as it requires regular supply of electricity.
(ii) Irrigation is not possible if the underground water level is low.
(iii) Excessive use of tubewell leads to lowering of groundwater level. [Any one point]
ll

P-44 G E OG R A P H Y - X
DRAMA
CHAPTER

MINERAL RESOURCES
17
WORKSHEET-29
Ans.1 Manganese is used to make steel tough, hard and rust resistant and that’s why it is an important raw
material for iron and steel industry.
Ans.2 Keonjhar, Mayurbhanj, Sambhalpur, Sundergarh, Cuttack and Koraput. [Any one] 
Ans.3 (i) Bauxite is mainly used in aircrafts, automobiles, shipping industry, household appliances, rail wagons,
coaches, etc.
(ii) Bauxite is found in Odisha, Goa, Gujarat, Maharashtra, etc.  [Any one] 
Ans.4 (i) Haematite
(ii) Magnetite
(iii) Limonite
(iv) Siderite.
Ans.5 (i) Iron ore – are used in Steel making / Slag for cement / sludge for fertilizer. [Any one use]
(ii) Bauxite- to extract aluminium/ light / used in aircraft/ use in automobiles
[ICSE Marking Scheme, 2015] 
Ans.6 (i) Aluminium
(ii) Used for manufacture of aircraft / utensils / wires. [ICSE Marking Scheme, 2016] 
Ans.7 (i) Cement - limestone
(ii) Aluminium - bauxite
(iii) Synthetics - oil or coal. [ICSE Marking Scheme, 2014] 
ll
WORKSHEET-30
Ans.1 Magnetite [ICSE Marking Scheme, 2016] 
Ans.2 Manganese is used in chemical industries for manufacturing bleaching powder.
Ans.3 Limestone is needed in the manufacture of iron and steel, cement, fertilizers / Fluxing material in
smelting of iron ore / glass manufacturing / manufacture of Chemicals – soda ash, caustic soda,
bleaching powder / Use in paper / sugar and aluminium. [Any two] [ICSE Marking Scheme, 2015] 

Ans.4 Any two of the following:


Raw material in the iron and steel industry / to make steel tough and rust proof
In the manufacture of black enamel
In the chemical industry
In the electrical industry
In the glass industry
In the battery industry
To form alloys. [ICSE Marking Scheme, 2014] 
Ans.5 Bauxite
Two uses of aluminium-
(i) Used in automobiles, aircraft
(ii) Packaging like cans, foil, etc.
Ans.6 (i) Bauxite
(ii) The largest deposits of Manganese are found in Balaghat in Madhya Pradesh. 
Ans.7 Odisha is the leading producer of manganese in India.
(i) It is used in manufacture of chemical and electrical equipments.

S O L U T I ON S P-45
(ii) It is used in chemical industries for manufacturing bleaching powder.
(iii) It is used in dry cell batteries. [Any two uses] 
Ans.8 Haematite, Magnetite, Limonite and Siderite are the different types of iron ore in India.
Magnetite is the best quality of iron ore.
ll
WORKSHEET-31
Ans.1 Kemmangundi [ICSE Marking Scheme, 2016] 
Ans.2 Haematite and Magnetite are mostly mined in India.
Two leading iron producing states are-
Karnataka, Odisha, Chhattisgarh, Goa and Jharkhand [Any two] 
Ans.3 (i) Odisha : Keonjhar
(ii) Jharkhand : Singhbhum
Ans.4 Two states in India where manganese is found are :
Odisha, Karnataka, Madhya Pradesh, Maharashtra, Andhra Pradesh, Goa, Telangana, Jharkhand and
Rajasthan. [Any two] 
One use of manganese :
(i) It is used in manufacture of chemical and electrical equipments.
(ii) It is used to manufacture coloured glass.
(iii) It is used in chemical industries for manufacturing bleaching powder.
(iv) It is used in dry cell batteries.
(v) Manganese is also used to manufacture vital enzymes for the metabolism of fats and proteins.
 [Any one use] 
Ans.5 Chhattisgarh, Jharkhand and Odisha. [Any two] 
Ans.6 (i) Two reasons why minerals are important :
1. Minerals are the source of raw materials and form the basis for industries.
2. They are the source of energy/power e.g. coal and petroleum.
(ii) Odisha : Keonjhar, Myurbhanj and Sambalpur
Chhattisgarh : Bailadillain Dantewada district and Durg district.
Ans.7 India is the largest producer of bauxite in South Asia.
Bauxite is considered an important mineral because
(i) It is rust resistant, strong and lightweight metal
(ii) A good conductor of electricity.
(iii) It is mainly used in aircrafts, automobiles, shipping industry, household appliances, rail wagons,
coaches, etc.
ll

P-46 G E OG R A P H Y - X
DRAMA
CHAPTER

ENERGY RESOURCES
16
 WORKSHEET-32
Ans.1 Raniganj coalfield.
Ans.2 Coal is widely used in the iron and steel industries.
Ans.3 Mumbai High. [ICSE Marking Scheme, 2016] 

Ans.4 Bituminous coal. [ICSE Marking Scheme, 2016] 


Ans.5 (i) Mumbai High.
(ii) Barauni refinery.
Ans.6 Anthracite, Bituminous and Lignite. [ICSE Marking Scheme, 2016] 
Ans.7 (i) Raniganj in West Bengal
(ii) Digboi in Assam [ICSE Marking Scheme, 2016] 
Ans.8 (i) Mathura / Guwahati / Digboi / Haldia / Koyali / Barauni / Kochin Chennai / Panipat / Mumbai /
Vishalapatnam
(ii) Jharkhand [ICSE Marking Scheme, 2015]
Ans.9 Bituminous is used for domestic purposes.
Burns slowly, no smoke, no ash, high heating value. [ICSE Marking Scheme, 2016] 
Ans.10 Iron and Steel Industry and the Sugar industry.
Ans.11 Coastal oil refinery- Mumbai and Vishakhapatnam
Inland oil refinery- Mathura and Barauni.
Ans.12 Bituminous coal is popular for domestic use. It contains 80% carbon in it and its calorific value is very
high.
Ans.13 Two off-shore oilfields of India are-
(i) Mumbai High.
(ii) Cambay Basin.
Ans.14 (i) 1.  Petroleum is widely used as fuel for transportation on land, on sea and in the air.
2.  It is also used for power generation.
(ii) One new oilfield in India- Aishwarya oilfield in Barmer district in Rajasthan.
One old oilfield in India- Digboi.
Ans.15 (i) As port cities are large consumers of oil and its products and by-products/to cut down transport costs
as most of the crude oil is imported.  [Any one point]
(ii) As petroleum is derived from the decomposition of organic matter under tremendous heat and
pressure.
(iii) As it has numerous uses-it can be converted into oil, gas and electricity/ used to generate power/ used
as fuel/used as a raw material in numerous industries like iron and steel, fertilizers and cement/ yields
valuable by-products like naphtha and phenol. [Any one point]
[ICSE Marking Scheme, 2015] 
Ans.16 (i) Coal - Jharkhand
(ii) Oil - Maharashtra
(iii) Manganese – Odisha [ICSE Marking Scheme, 2014] 
Ans.17 (i) Anthracite has a heating value, high caloric value and is smokeless and is thus good for domestic
purpose.
(ii) Oil refineries are located close to oil fields or near ports to avoid transportation of mineral oil to the
interior places of the country as it is highly inflammable and risky.
(iii) It is so because to manufacture iron and steel, it is used as the important source of energy and is also
used as a raw material in the chemical and other industries.
ll
S O L U T I ON S P-47
 WORKSHEET-33
Ans.1 Jharia in Jharkhand and Raniganj in West Bengal. 
Ans.2 Maharashtra is the largest producer of mineral oil. 
Ans.3 Mathura Refinery and Mumbai Refinery.
Ans.4 Coal is considered the most important industrial power resource because coal is found in abundance
and 67% of energy needed in our country is met by coal.
Ans.5 Two advantages of natural gas-
(i) It can be stored safely and can be transported efficiently through pipelines, cylinders, etc.
(ii) It is reliable and is conveniently used for cooking and for running many appliances.
(iii) It is cheaper and cleaner than petrol or diesel. [Any two points]
Ans.6 West Bengal and Jharkhand are the two states with large deposits of coal.
Raniganj in West Bengal and Jharia in Jharkhand.
Ans.7 Lignite is a soft brown combustible sedimentary rock and has a carbon content of only 40%.
Lignite is mined in Neyveli in Tamil Nadu.
Ans.8 Jharkhand is an important coal producing state in India.
Jharia coal-mine is located in that state.
Ans.9 Two important oil fields in India are-
(i) Digboi in Assam.
(ii) Cambay Basin in Gujarat.
Ans.10 Two main drawbacks of coal found in India-
(i) Greenhouse Gas Emissions is considered as one of the biggest drawbacks of coal energy.
(ii) It also results in harmful emission of substances like Mercury and Sulfur Dioxide which cause health
hazards amongst the population of that area.
(iii) Uneven distribution of coal.  [Any two points] 
Ans.11 Gondwana Coal fields and Tertiary coal fields are the two coal fields in India.
Gondwana Coalfields- West Bengal
Tertiary Coalfields- Assam
Ans.12 Two products of an Oil refinery are Gasoline, LPG, naphtha, diesel, kerosene, fuel, lubricating oils,
bitumen, petroleum and coke. [Any two products]
Oil refinery along the coast- Trombay and Vishakhapatnam.
Oil refinery away from the coast- Mathura and Barauni.
Ans.13 Regions which has natural gas deposits are Mumbai High, Assam, Rajasthan, Tamil Nadu and Tripura.
 [Any one region]
Two uses of Natural Gas-
(i) It is mainly used as a fuel for generating electricity and heat.
(ii) Natural gas in compressed form is used as fuel for vehicles which is known as CNG (Compressed
Natural Gas).
Ans.14 Oil refineries are located close to an oil field or in a coastal city because-
1.  To minimise the cost of transport
2. To avoid transportation of mineral oil to the interior places of the country as it is highly inflammable.
One oil refinery in the private sector is The Reliance Petroleum Limited at Jamnagar in Gujarat.
Ans.15 (i) Oldest coal field- Raniganj in West Bengal
Largest coal field- Jharia in Jharkhand
(ii) Two raw materials derived from coal-
1. Coke
2. Tar
Ans.16 (i) Two uses of mineral oil-
1. It is used as a fuel and as a lubricant.
2. It is also used in the chemical, pharmaceutical, textile and many other industries.

P-48 G E OG R A P H Y - X
(ii) Old mineral producing area- Assam
New mineral producing area- Mumbai High 
Ans.17 Anthracite, Bituminous, Lignite and Peat are the four types of coal.
Bituminous coal is the best for industrial purposes. It is an important ingredient in iron and steel
smelting in blast furnaces.
Ans.18 Three disadvantages of conventional sources of energy are-
(i) These are non-renewable sources of energy.
(ii) They are non-environment friendly.
(iii) These are exhaustible sources.
Ans.19 (i) An impervious rock, that prevents material from leaking out.
(ii) Organic material composed of dead animals and plants in sufficient supply.
(iii) Right pressure and temperature to allow the organic material to be transformed into hydrocarbons.
(iv) Millions of years of time.
Two advantages of using natural gas-
(i) Natural Gas is considered to be environment friendly.
(ii) It can be stored safely and can be transported efficiently through pipelines, cylinders, etc.
(iii) It is reliable and is conveniently used for cooking and for running many appliances.
[Any two points] 
ll
 WORKSHEET-34
Ans.1 Multi-purpose projects are vital because these projects are conducive to the economic development of
the country, are boosting agriculture through irrigation, prevent flooding and generate electricity.
Ans.2 Two advantages of Hydro-Electric power over Coal and Petroleum-
(i) Hydro-Electric power produces electricity from water and thus non-pollutant.
(ii) It is inexhaustible and renewable source of energy.
Ans.3 Bhakra-Nangal Project has been built on the river Sutlej.
Punjab, Haryana and Rajasthan are benefitted by this project.
Ans.4
Dam Barrage
1. It is built for storage of water in a reservoir 1. It is built to divert the river water.
to raise the level of water.
2. It stores surplus flood water and 2. There is no storage of water; canals take water
distributes additional water through directly from the rivers..
power and irrigation.

Ans.5 Two hazards of constructing dam :
(i) Ecological impact due to the disturbance in the ecology of water.
(ii) Human displacement from the place where they have been living for long period of time.
Ans.6 Three advantages that hydroelectric power has over thermal power are :
(i) Hydropower is generated from the force of running water while thermal power is produced by
burning coal, petroleum or natural gas.
(ii) Hydel power is a clean source of energy and do not cause pollution while thermal produces air and
dust pollution.
(iii) Hydel power is inexhaustible and renewable sources of energy while thermal power is non-renewable
sources of energy.
Ans.7
Thermal Power Project Hydel Power Project
1. It is produced by burning coal, petroleum, 1. It is generated by storing water.
natural gas, etc.

S O L U T I ON S P-49
2. The initial cost of generating plants is 2. The initial cost of generating plants is costly
cheaper but maintenance costs are high but maintenance costs are low and less labour
and need more labour. needed.
3. It causes pollution. 3. It is clean and non-polluting.
4. These are non-renewable and exhaustible 4. These are renewable and inexhaustible sources
sources of energy. of energy.

ll
 WORKSHEET-35
Ans.1 Methane gas is generated in bio-gas plants.
Biogas is a non-polluting, cheap and clean source of energy.
Ans.2 Solar Energy is the most widespread non-conventional source of power.
India is a tropical country and receive ample amount of solar energy throughout the year. It is also
non-polluting and inexpensive.
Ans.3 Bio-gas is increasingly being used because-
(i) Biogas is a cheap and non-polluting.
(ii) It generates employment in the rural areas.
(iii) It is renewable source of energy.
(iv) It significantly lowers the greenhouse effects on the earth’s atmosphere. [Any two points] 
Ans.4 Nuclear power generation carried out on a small scale in India because :
(i) It is expensive.
(ii) There is shortage of raw materials like uranium and thorium.
(iii) There is a problem of radioactive waste disposal.
ll

P-50 G E OG R A P H Y - X
DRAMA
CHAPTER

AGRICULTURE I AND II
19
 WORKSHEET-36
Ans.1 (i) Inadequate use of manures and fertilizers, negligence of crop rotation, use of poor quality seeds,
inadequate water supply, etc. leads to low productivity.
(ii) Use of simple and old agricultural tools, use of no or less machines for ploughing, sowing, irrigating,
pruning, harvesting and threshing results in low yield.
Ans.2 The consolidation of landholdings helped the farmers in the following ways :
(i) It saved time, energy and money in moving from one farm to another.
(ii) Farmers feel encouraged to spend money on the improvement of his land. [Any one point] 
Ans.3 – Provides food for our growing population
(i) Provides fodder for livestock
(ii) Provides employment
(iii) Supplies raw material for agro-based industries
(iv) Gives rise to industries related to agriculture, like pesticides, fertilizers, farming tools
(v) Earns foreign exchange through export. [ICSE Marking Scheme, 2016] 
[Any two]
Ans. 4 Farm fragmentation reduces the size of the farm on which it is not possible to use harvesters or other
farm machinery and modern methods of agriculture. Because of poverty the farmers cannot afford
good quality seeds, fertilizers or pesticides or farm implements and so the yield is low.
[ICSE Marking Scheme, 2014] 
Ans.5 Problems of Agriculture in India-
(i) Lack of adequate irrigation facilities and dependence on monsoon.
(ii) The land holdings are uneconomic due to their small size and as such the yields are low.
(iii) Agriculture is becoming mechanized and requires huge capital investments to purchase machineries,
fertilizers, pesticides and high yielding variety seeds. The Indian farmers are poor to buy all these
materials.
(iv) A majority of Indian farmers are still dependent on the primitive and poor techniques of producing
crops. [Any two points] 
ll
 WORKSHEET-37
Ans.1 (i) Plantation farming is an extensive system of agriculture in which single cash crop is cultivated on a
large scale in an estate.
(ii) Crops like tea, coffee, rubber, spices, etc. are grown under plantation farming mainly for profit.
Ans.2
SUBSISTENCE FARMING COMMERCIAL FARMING
1. It is a self-sufficient farming in which the 1. 
It is a farming where crops are grown and
farmers grow enough food to feed himself animals are reared for sale in the market for
and his family. commercial purposes.
2. The farmer uses simple and primitive tools 2. 
It largely depends on machines, uses HYV
with traditional method of agriculture. seeds, chemical fertilizers, pesticides and
insecticides to obtain higher yield.

Ans.3 Mixed Farming is a combination of growing crops and rearing of cattle simultaneously. The main
benefit of this type of farming is that it ensures a steady income for the farmers because if any one
business or farming fails, the other means can support.

S O L U T I ON S P-51
Ans4. (i) Shifting cultivation involves a patch of forest which is cleared, trees are cut down and the stumps set
on fire. The patch is then cultivated for a few years and when the soil becomes infertile, the cultivator
moves to a fresh piece of land and repeats the same procedure.
(ii) The bud from a good rubber tree is grafted on the seedling of a new rubber tree, Once the bud starts
sprouting the shoot from the seedling is cut down and the bud then grows into a tree with all of the
traits of the mother tree. [ICSE Marking Scheme, 2016] 
Ans.5 Any two of the following:
Huge capital investment
Large land holding
Labour intensive
Single crop or monoculture
Crop grown usually for export
Crops grown as plantation crops are tea, coffee, rubber and spices. [ICSE Marking Scheme, 2016] 
Ans.6
INTENSIVE FARMING EXTENSIVE FARMING
1. Intensive farming is a system of farming that 1. Extensive farming uses machinery and
involves higher input of labour, increased use scientific methods to produce large quantity
of fertilizers, pesticides, high quality seeds, of crops.
etc.
2. It is labour intensive farming. 2. It is highly capital intensive.
Ans.7 Characteristics of plantation farming-
(i) Plantation farming is an extensive system of agriculture in which single cash crop is cultivated on a
large scale in an estate.
(ii) Crops like tea, coffee, rubber, spices, etc. are grown under plantation farming mainly for profit.
(iii) This type of farming is practiced in vast lands extending from a few hectares to thousands of hectares.
(iv) Modern methods, techniques and machineries are used for growing crops.
(v) Huge capital is invested in buying machineries, fertilizers, pesticides and building factories for
processing of crops.  [Any two points]
One important plantation crop-Tea.
Ans.8
SHIFTING AGRICULTURE PLANTATION AGRICULTURE
1. Shifting agriculture is also known as “slash 1. Plantation farming is an extensive system
and burn method”. It is a primitive method of of agriculture in which single cash crop is
farming in which a patch of forest is cleared by cultivated on a large scale in an estate.
felling trees or by burning the trees.
2. The crops grown in this type of farming are- 2. Crops like tea, coffee, rubber, spices, etc. are
maize, millets, barley, buckwheat, root crops, grown under plantation farming mainly for
rain fed rice and vegetables. profit.
Ans.9 Two main features of Subsistence Agriculture :
(i) Subsistence farming is a self-sufficient farming in which the farmers grow enough food to feed himself
and his family.
(ii) The farmers have small land and do not use fertilizers and thus the yield is low.
(iii) The output is mostly for local requirements with little or no surplus trade.
(iv) The land holdings are small and scattered.
(v) The farmer uses simple and primitive tools with traditional method of agriculture.
[Any two points] 
ll
 WORKSHEET-38
Ans.1 Sowing and harvesting are the two methods of growing wheat.

P-52 G E OG R A P H Y - X
Ans.2 Millets are known as dry crops because they can grow in areas of low rainfall ranging from
50-100 cms.
Ans.3 Pulses are high in protein and are an important vegetarian diet. [ICSE Marking Scheme, 2015] 
Ans.4 In this method after ploughing, the seeds are scattered all over the field by hand before the onset of
monsoon.
Ans.5 (i) Transplantation is the process in which seedlings are first grown in nurseries and after 4 to 5 weeks
when the saplings attain a height of 25 to 30 cm they are transplanted to prepared rice fields.
(ii) Transplantation enables to select only healthy seedlings for the plants.
Ans.6 Drilling- In this method the seeds are sown in the furrows with the help of a drill made of a bamboo.
Ans.7 Pulses are grown as rotation crops as they are leguminous crops that fix atmospheric nitrogen in the
soil and increase the natural fertility of the soil.
Ans.8 Kharif Season and Rabi Season are the two main crop seasons in India.
Kharif Crops : Rice, jowar, sugarcane, bajra, ragi, maize, cotton and jute.
Rabi Crops : Wheat, barley, rapeseed, linseed, gram, peas, mustard.
Ans.9 (i) Rice needs flooded fields and the subsoil of clay does not allow the water to percolate.
(ii) By growing rice in nurseries, we save water, and the plants can be transplanted in rows to facilitate
the use of pesticides and harvesting. [ICSE Marking Scheme, 2016] 

Ans.10 Rabi Crop Kharif Crop


Sown in Oct-Nov and harvested in Mar-April / Sown in June and harvested in Oct-Nov/
Winter crop Summer crop
[ICSE Marking Scheme, 2015] 
Ans.11 (i) West Bengal, Punjab, Uttar Pradesh and Andhra Pradesh. [Any two states]
(ii) Advantages of growing rice in nurseries :
1. It enables deeper penetration of the roots in the soil.
2. Less wastage of seeds.
3. It minimizes weed pressure by resowing.
4. It gives higher yield. [Any two points] 
Ans.12 Temperature required- 10°C-15°C.
Rainfall- 50 cm to 100 cm
Soil- well-drained loams and clay loams.
Ans.13 (i) Rice. It enables deeper penetration of the roots in the soil and gives higher yield.
(ii) 1. Temperature- 18°C - 32°C
2.  Rainfall- 150 cm - 300 cm
3.  Soil- Deep fertile clayey or loamy soils. 
Ans.14 (i) (1) Kharif crops are sown in the months of June and July and (2) harvested in September and October.
(ii) Cotton.
Ans.15 Wheat is grown in the Rabi season in northern India.
Rainfall- 50 cm to 100 cm
Soil- Alluvial soil or well-drained loamy and clay loams soil.
Ans.16 Wheat is a Rabi crop because :
1. Sown in October- November and harvested in January in south, by March-April in north.
2. Temperature of 10°C-15°C is suitable for sowing and 20°C-25°C during harvest and rainfall
between 50 cm to 100 cms.
Ans.17 1. Temperature- 18°C - 32°C
2.  Rainfall- 150 cm - 300 cm
3.  Soil- Deep fertile clayey or loamy soils.
Ans.18 (i) Transplantation : In transplantation, seedlings are first grown in nurseries and after 4 to 5 weeks
when the saplings attain a height of 25 to 30 cm they are transplanted to prepared rice fields.
(ii) Broadcasting : In this method after ploughing, the seeds are scattered all over the field by hand before
the onset of monsoon.

S O L U T I ON S P-53
Ans.19 Rice Wheat
Kharif crop Rabi crop
18°C - 32°C; 150 to 300 cm of rain 10°C- 15°C; 50 to 100 cm of rain
Requires alluvial soil with a subsoil of clay to Loamy soil
allow water to stagnate
Sown by broadcasting, dibbling, drilling or Sown by drilling
transplantation
Harvested by hand Harvesters used
[ICSE Marking Scheme, 2014] 
Ans.20 Three aspects of Japanese method of cultivation-
(i) Use of High Yielding Variety (HYV) of seeds.
(ii) Saplings are sown in the nursery and raised in the nursery beds for 4-5 weeks.
(iii) Manure is extensively used to enhance the yield. 
Ans.21 Importance of Pulses :
(i) Pulses form an important part of the Indian diets because they are full of protein.
(ii) Pulses are grown as rotation crops as they are leguminous crops that fix atmospheric nitrogen in the
soil and increase the natural fertility of the soil.
(iii) Pulses are good cattle fodder too.
Ans.22 Major crop seasons are :
(i) Kharif Crops : The cropping season is from July to October during the south west monsoon. Rice,
jowar, sugarcane, bajra, ragi, maize, cotton and jute are some of the important kharif crops.
(ii) Rabi Season : The cropping season is from October to March. Crops like wheat, barley, rapeseed,
linseed, gram, peas, mustard, potatoes, etc. are grown as Rabi crops.
Ans.23 (i) It enables to select only healthy seedlings for the plants.
(ii) Less wastage of seeds.
(iii) It minimizes weed pressure by resowing.
(iv) It gives higher yield.
ll

P-54 G E OG R A P H Y - X
DRAMA
CHAPTER

AGRICULTURE III AND IV


1
10 [CASH CROPS (1) AND (2)]AND II

 WORKSHEET-39
Ans.1 One problem faced by the sugarcane cultivators in India-
(i) Sugarcane is a soil-exhausting crop and thus need good amount of fertilizers which increases the cost
of production.
(ii) The location of sugar mills are far from the fields, thus, a delay of more than 24 hours results in the
reduction of sucrose content in the canes. [Any one point] 
Ans.2 Ratooning is a method in which during harvesting of sugarcane plant, the roots and the lower parts
of the plant are left uncut to give the ratoon or the subtle crop.
It is associated with Sugarcane.
Ans.3 Oil cake is the residue of the oil seeds from which the oil has been extracted.
[ICSE Marking Scheme, 2016] 
Ans.4 As the mills are closer to the fields hence there is less loss of sucrose content/ use of better quality
cane/larger farms, hence mechanised farming is possible/co-operative farming is practised, ensuring
a better yield with better seeds, increased use of better fertilizers, better irrigation method and better
crop protection measures/Frost free growing season/ tapering shape of peninsular- Sea breeze/Longer
crushing season/regur soil responsible for better yield.  [Any one point] 
[ICSE Marking Scheme, 2015]
Ans.5 Two non-edible oilseeds grown in India :
(i) Linseed : It is used in paints, varnishes, linoleum, oil cloth, printing inks. Linseed oil was often used
for tempering wood (especially new bats—cricket, hockey etc).
(ii) Castor seed : Its oil is used in lubricants, soaps, inks, varnishes and linoleum.
[Any one use of each] 
Ans.6 (i) Ratooning
(ii) Oil cake
(iii) Coagulation [ICSE Marking Scheme, 2015] 
Ans.7 (i) Sett Method and Ratoon Method.
(ii) Sugarcane is a labour intensive crop and needs skilled labour for cutting the stalks from the ground
carefully and efficiently.
(iii) So that sugarcane can be processed within 48 hours of cutting to preserve the sucrose content.
Ans.8 (i) 1.  The mills are closer to the fields hence there is less loss of sucrose content.
2.  Use of better quality cane and longer crushing season.
3.  Grown in large farms and hence mechanised farming is possible.
4.  Frost free growing season. [Any two points]
(ii) One advantage of ratoon cropping-
This method is inexpensive as no preparation of the field is required.
One disadvantage of ratoon cropping-
Ratoons produce low quality crop as with successive year the canes are thinner with low sucrose
content.
Ans.9 (i) Geographical conditions necessary for the cultivation of groundnuts are :
1.  Temperature- 20°C to 25°C
2.  Rainfall- Between 50 cm-100 cm
3.  Black soil, sandy loams and loamy soil are ideal for the crop.
(ii) The climatic condition that adversely affects the groundnut crop are-
1.  Prolonged drought
2.  Continuous rains

S O L U T I ON S P-55
3.  Stagnant water
4.  Frost
(iii) Linseed and Castor.
Ans.10 Gujarat is the largest producer of Castor seed.
Important use of its oil : Used in paints, varnishes, printing ink, etc.
Important use of oil cake : Used as animal fodder. 
Ans.11 Problems of Sugarcane Cultivation :
(i) Sugarcane is a soil-exhausting crop and thus need good amount of fertilizers which increases the cost
of production.
(ii) In India the yield per hectare is extremely low as compared to other countries of the world.
(iii) Sugarcane has a short crushing season normally from 4 to 7 months in a year which results in financial
problems for the industry as the mills and the workers remain idle.
(iv) The location of sugar mills are far from the fields, thus, a delay of more than 24 hours results in the
reduction of sucrose content in the canes.
(v) Sugarcane is an annual crop but the land available for sugarcane is less as compared to other crops,
thus, the farmers are unable to cultivate any other crop.
(vi) The production cost of sugarcane in India is the highest in the world due to uneconomic process of
production, inefficient technology and heavy excise duty.
(vii) Small and uneconomic size of mills.
(viii) Old and obsolete machinery are used in most of the Indian sugar mills and needs rehabilitation.
(ix) Sugar industry is facing competition with gur and khandsari since Khandsari industry is free from excise
duty and can offer higher prices of cane to the cane growers.
(x) Sugarcane cultivation needs good amount of water but lacks irrigation facilities.
(xi) The government has fixed prices for the sugarcane farmers which is not profitable for them.
 [Any two points] 
Role of Government in Solving Farmer’s Problems-
(i) The government has set up a number of Cooperative Societies.
(ii) To develop various means of irrigation to provide regular supply of water to the sugarcane fields.
(iii) To provide adequate and timely loans to farmers on easy terms so that they can buy farm machinery
and other agricultural items.
(iv) To educate farmers with latest farming techniques and help farmers through specially developed
programmes on radio and television.  [Any two] 
ll
 WORKSHEET-40
Ans.1 Any one of the following:
Oil of the oil seeds is useful, gives edible oil and raw material for products like paints, varnishes, soaps etc.
Provides employment
The oil cake used for fodder
Exported and so earns foreign exchange. [ICSE Marking Scheme, 2016] 
Ans.2 (i) After harvesting of sugarcane plant, the roots and the lower parts of the plant are left uncut to give the
ratoon or the subtle crop.
(ii) The successive crops that grow from the left out subtle is called the Ratoon. 
Ans.3 Oilcake is used as animal fodder and is also used as good manure in the farms. 
Ans.4 Gujarat is an important producer of groundnuts.
Ans.5 1.  Temperature- Grows best in areas with temperature between 20°C and 24°C.
2.  It requires 100 cm-150 cm of rainfall throughout the year.
3.  The crop grows well in well drained rich alluvial, heavy loams or lava soil. 
Ans.6 Castor seed is a non-edible oil seed.
Its oil is used in lubricants, soaps, inks, varnishes and linoleum. 

P-56 G E OG R A P H Y - X
Ans.7 (i) Due to climatic fluctuations and low market value, the farmers prefer other commercial crops.
(ii) Lack of high yielding varieties, irrigation facilities and chemical fertilizers. 
Ans.8 (i) Groundnut is mainly used for the manufacture of hydrogenated oil and is used in making margarine,
soap, medicines, cooking oil, etc.
(ii) It is eaten raw, roasted and salted.
(iii) Its oilcake is used as cattle fodder.
(iv) Being a leguminous plant it plays an important role in crop rotation. [Any 3 points] 
Ans.9 Linseed is obtained from the flax plant.
Commercial Uses of its Oil-
(i) It is used in paints, varnishes, linoleum, oil cloth, printing inks.
(ii) It is often used for tempering wood (especially new bats—cricket, hockey etc).
(iii) Oil cake is used as cattle fodder.
ll
 WORKSHEET-41
Ans.1 To allow excess water to drain off as tea cannot tolerate stagnant water.
[ICSE Marking Scheme, 2015] 
Ans.2 It means to reduce the moisture content in the tea leaves, make the leaves soft and to allow the flavour
compounds to develop.
Ans.3 Tea bushes are pruned to maintain the height of the bush to facilitate the plucking of leaves and to
have new shoots bearing plenty of soft leaves.
Ans.4 New alluvium fertile soil in the Ganga delta region is most suitable for jute cultivation.
Ans.5 Ginning is a process of separation of cotton fibre from the cotton seed.
Ans.6 (i) Karnataka, Kerala and Tamil Nadu.
(ii) Temperature- Between 15°C and 28°C.
Rainfall- Between 150 cm and 200 cm of annual rainfall. Stagnant water is harmful.
Soil- Well drained, rich friable loams containing a good deal of humus and minerals like iron and
calcium are ideal for coffee cultivation.
Ans.7 (i) Pruning is an essential part of tea cultivation as it helps in maintaining the proper shape of tea bush
to a height of about one metre to facilitate the plucking of leaves.
(ii) The coffee beans are roasted at temperatures of about 99°C and then ground into coffee powder.
Roasting gives the brown colour, aroma and taste.
Ans.8 Cotton Jute
Temp : 20°C - 30°C Temp : 21°C- 35°C
Rain : 50 cm- 120 cm Rain : 150 cm - 250 cm
[ICSE Marking Scheme, 2015] 

Ans.9 (i) 21°C to 30°C; 50 to 75 cm of rain; black soil.


(ii) The new tea plant has all the qualities of the mother plant from which the cutting is taken and
sown.  [ICSE Marking Scheme, 2014] 
Ans.10 (i) Retting : It is a microbiological process which loosens the outer bark and facilitates removal of the
fibre from the stalk.
(ii) Ginning : It is a process of separation of cotton fibre from the cotton seed. 
Ans.11 Uttar Pradesh produces short staple cotton.
Temperature- 21°C and 30°C.
Rainfall- 60 cm to 120 cm.
Alluvial soil.
Ans.12 Gujarat leads in cotton cultivation.
Two climatic factors which affect the cotton cultivation adversely are-
1. Cotton plant is extremely sensitive to frost.

S O L U T I ON S P-57
2. Rainfall during the cotton ball opening and harvesting periods is harmful for the plants.
Ans.13 West Bengal is the leading producer of Jute.
Rainfall- 150 cm -200 cm.
Soil- New Alluvial fertile soil in the Ganga-Brahmaputra delta.
Ans.14 (i) Ginning : It is a process of separation of cotton fibre from the cotton seed.
(ii) Arabica : An evergreen shrub or tree yielding seeds that produce a high quality coffee.
Ans.15 Temperature- 24°C to 30°C.
Rainfall- 150 cm- 500 cm.
Soil- Well drained, deep friable loamy soil.
Ans.16 (i) Temperature- 21°C and 30°C with atleast 200 frost free days.
(ii) Rainfall- 60 cm to 120 cm.
(iii) Soil- Deep black clay soil.
Ans.17 (i) Assam / West Bengal / Kerala / Tamil Nadu
(ii) Mostly women harvest the tea because it involves fine plucking, (two leaves and a bud) and women
labourer can patiently pluck the same.
(iii) Two geographical conditions are:
Temperature – 24°C to 30°C
Rain – 150 cm to 350 cm
Soil – well drained deep friable loams, rich in organic matter.
Laterite soil [Any two] 
[ICSE Marking Scheme, 2016]
Ans.18 (i) Jute has to be retted to facilitate the removal of the fibre from the stalk.
(ii) Tea is grown on hill slopes because water logging at the roots of the plant is injurious and so it prevents
from water stagnation.
(iii) The coffee plant cannot stand direct sunrays and is thus grown under shady trees such as silver oak
and banana trees.
Ans.19 (i) Coffee Arabica and Coffee Robusta.
(ii) The coffee plant cannot stand direct sunrays and is thus inter-planted with orange trees, cardamom
and pepper vines.
(iii) Karnataka.
Ans.20 (i) The two main varieties of coffee are-
1.  Arabica
2.  Robusta
(ii) 1. Dalap and Silver Oak are grown in coffee estates as shade trees as they protect the coffee plants
from the direct rays of the sun.
2.  Orange and Plum are grown to provide extra income to the farmers.
Ans.21 Tea is considered a labour intensive crop because most of the tea cultivation activities are done
manually by the skilled and unskilled farmers and labourers like sowing, pruning, plucking, etc.
Two states in India where tea is widely grown are : Assam and West Bengal.
ll

P-58 G E OG R A P H Y - X
DRAMA
CHAPTER

AGRO-BASED INDUSTRY
1
11
 WORKSHEET-42
Ans.1 On the basis of the nature of products, the industries are classified into :
(i) Heavy Industries : Ship building, machinery manufacturing, iron and steel.
(ii) Light Industries : Sewing Machines, cycles, electronic goods.
Ans.2
Public Sector Industry Private Sector Industry
1. These industries are owned and managed by 1. These industries are owned and managed
Central Government or State Government privately by individuals or group of
which includes industries of public utility. individuals.
2. E.g. Post and telegraph, railways, oil refineries, 2. E.g. Industries like Reliance India Limited,
heavy engineering industries, defence Wipro, Infosys, TCS, etc.
establishments, Bharat Heavy Electricals
Limited (BHEL), Steel Authority of India
Limited (SAIL), Gas Authority of India Limited
(GAIL), etc.

Ans.3 (i) Agro-based industries provide agricultural raw materials to the industries.
(ii) These industries are good foreign exchange earner.
Ans.4
BASIC INDUSTRIES CONSUMER INDUSTRIES
1. These Industries depend on other industries 1. These industries process the basic raw materials
for their manufacturing. into primary goods for direct use by the
consumers.
2. Iron and steel industry and petroleum 2. Textile industry, Sugar Industry, Automobile
industry are major basic industries. industry are some main consumer industries.

Ans.5 (i) It provides employment to a large number of people.


(ii) It is a subsidiary occupation since agriculture alone cannot support livelihood to all rural people.
Ans.6 (i) Industry should be set up close to the availability of the raw material.
(ii) Nearness to the market.
(iii) Proper transportation infrastructure for transporting the raw materials to the industry.
(iv) Cheap and regular supply of power.
Ans.7 (i) India is an agricultural country and through industrialisation the development of agriculture can be
initiated.
(ii) Only agriculture cannot generate employment and thus establishment of industries can generate
employment opportunities on a large scale.
(iii) The development of industries producing capital goods i.e. machines, equipment etc. enables a
country to produce a variety of goods in large quantities and at low costs and make for technological
progress.
(iv) Through industrialisation infrastructures like railways, roadways, dams, etc. can be constructed which
can enhance the future growth of Indian economy.
(v) Industrialisation is a necessity for country’s security because only through industrial development
self-reliance in defence can be achieved where she can produce her own defence materials.
(vi) Expansion of industries in the backward regions of India is needed to counter the regional imbalance.
 [Any three points] 

S O L U T I ON S P-59
Ans.8 Cottage industries are organised by individuals with private resources and with the help of their
family members and their skills e.g. weaving, handloom, carpet industry, etc.
Cottage industries are important because :
(i) It provides employment to a large number of people.
(ii) It is a subsidiary occupation since agriculture alone cannot support livelihood to all rural people.
ll
 WORKSHEET-43
Ans.1 Cotton textile industry is called an agro-based industry because it requires cotton as its raw material
which is an agricultural product.
Ans.2 Karnataka.
Ans.3 (i) Competition from artificial silk which is cheap and of better quality.
(ii) Import of cheap and alternative textiles from China and other Asian countries.
(iii) Use of outdated manufacturing technology, primitive and unscientific ‘reeling’ and ‘weaving’
techniques, etc.
(iv) The price fluctuation of raw silk affects the weavers and the industry.
(v) Lack of new technologies and modern power looms is affecting the growth of production.
[Any One] 
Ans.4 (i) Regular supply or proximity to raw material.
(ii) Favourable climatic conditions.
(iii) Good network of road and rail transportation within the country and sea routes for the international
market.
(iv) Location of major ports facilitates the export and import facilities.
(v) Availability of cheap and skilled labours. [Any two points] 
Ans.5
Mineral-based Industry Agro-based Industry
1. These industries use minerals both metallic 1. These industries depend on the raw
and non-metallic as raw materials. materials produced by the agricultural
sector.
2. These industries produce both consumer and 2. They mostly produce consumer goods.
value based goods.
3. These industries supply their raw material to 3. The major Agro-based industries are cotton,
iron and steel industry, heavy engineering and sugar, jute, tea, coffee, etc.
machine tool industry, cement industry, etc.

Ans.6 (i) The maritime climate of Maharashtra which is free from loo and frost.
(ii) The availability of black soil which is well drained and more fertile than alluvial soil.
(iii) Excellent transport facilities in Maharashtra has given it an advantageous position in relation to export
markets.
(iv) The sugar factories are located close to the sugarcane farms which prevent the loss of sucrose content
due to minimum transportation time.
(v) The farmers have new machinery and crushing devices which ensures high yield.
[Any two points] 
Ans.7 Fluctuation in availability of raw material as it is agricultural based / outdated machines / frequent
power cut. [ICSE Marking Scheme, 2016] 
Ans.8 Availability of raw material as it is a jute producing area / Plenty of clean water for retting / availability
of labour and transport facility. [Any two] 
[ICSE Marking Scheme, 2016]
ll

P-60 G E OG R A P H Y - X
 WORKSHEET-44
Ans.1 (i) As it requires cotton, an agricultural product as its raw material.
(ii) It is more widespread than the jute textile industry because cotton is grown all over the country/
whereas jute is cultivated mainly in the east of India/ there continues to be a great demand for cotton
all over the country/hence mills are set up everywhere; whereas the demand for jute is declining as it
is facing stiff competition from synthetic material like nylon and plastic/ India being a tropical country
more demand for cotton fabrics/ affordable by mass.  [Any one point] 
[ICSE Marking Scheme, 2015]
Ans.2 (i) One important point of similarity between the woollen and silk industry is that both use animal
products as raw materials/ both are small-scale/cottage industries. [Any one point]
(ii) Woollen industry- Punjab; Silk Industry- Karnataka [ICSE Marking Scheme, 2015] 
Ans.3 Wool - Jammu & Kashmir / Punjab / Haryana / Himachal Pradesh / Uttar Pradesh Silk - Karnataka /
Andhra Pradesh / Tamil Nadu / West Bengal / Bihar / Jharkhand / Assam.
[ICSE Marking Scheme, 2014] 
Ans.4 Any two of the following:
(i) Water from the Ganga and her distributaries Power from DVC
Kolkata is a port
Well connected by road and rail
Labour is easily available from West Bengal, Odisha, Jharkhand.
(ii) India is a tropical country, so the demand for wool is not as much as for cotton.
Woollen goods are not as profitable as cotton, so more investment in the cotton industry. Woollen
goods are not as affordable as cotton The sheep do not have a thick or good quality fleece Competition
from synthetic material. [ICSE Marking Scheme, 2014] 
Ans.5 (i) The rearing of silk worms for the production of silk is known as Sericulture.
(ii) Karnataka is famous for silk industry because-
1. It has favourable climate for rearing silkworms.
2. The state-owned Channapatna has a capacity of numerous spindles.
3. There are nurseries, silk farms and licensed seed distributors.
4. The decentralized sector has many powerlooms and handlooms. [Any two points] 
Ans.6 (i) Tasar Silk
(ii) Bagasse
(iii) Sericulture
Ans.7 (i) (a)  Regular supply or proximity to raw material.
(b) Favourable climatic conditions specially the humid climate.
(c) Good network of road and rail transportation within the country and sea routes for the
international market.
(d) Location of major ports facilitates the export and import facilities.
(e) Availability of cheap and skilled labours.
(f) Mumbai is a centre of financial and commercial resources.
(g) Electricity is supplied by the Tata Hydroelectricity system in the Western Ghats to Mumbai.
 [Any two points]
(ii) Ahmedabad and Coimbatore. 
ll

S O L U T I ON S P-61
 WORKSHEET-45
Ans.1 (i) Climatic condition of Karnataka is suitable for mulberry tree and so sericulture can be practiced.
(ii) Tassar, Eri, Muga [Any two]
(iii) Uttar Pradesh – Mirzapur / Pratapgarh / Shahjahanpur  [Any one]
Tamil Nadu – Coimbatore / Salem / Tanjavur / Tirunelveli [Any one]
  [ICSE Marking Scheme, 2016] 
Ans.2 (i) India is a tropical country and so demand of woollen is low / Poor quality of indigenous wool /
Woollens are expensive so less demand / Winter is severe only in north so there is small market.
(ii) Amritsar, Ludhiana, Dhariwal. [ICSE Marking Scheme, 2016] 
Ans.3 (i) Problems of the sugar industry: poor quality of cane/out-dated machinery/fluctuating supply of raw
material/low profit margins as prices are fixed by the govt./inadequate use of by-products, increasing
production costs/mills are located far from fields, leading to loss of sucrose/ the seasonal nature of the
industry, increasing production costs/small size of farms/inadequate use of fertilizers.
 [Any two problems]
(ii) By- products- bagasse, molasses, pressmud.  [Any two points]
[ICSE Marking Scheme, 2015]
Ans.4 (i) Introduction of artificial silk which is more durable / Easy to maintain / crease proof / cheaper to
produce /Need for modern power looms for increase in production/ No systematic testing and
grading of silk/ changes in price of raw silk affect both growers and the industry.  [Any one points]
(ii) Two products of the jute industry- carpets/ wall- hangings/pot holders fabrics/sandals/hand-bags.
 [Any two points]
(iii) As they are cheaper/are more durable /are moth resistant/ not dependent on agriculture for raw
materials / Chemicals are easily available. [Any one points]
[ICSE Marking Scheme, 2015] 
Ans.5 Any three of the following :
Agro-based industry, therefore the supply fluctuates
Unproductive workers
Outdated machinery and methods of processing jute
Competition from synthetic materials
Competition from countries like Bangladesh. [ICSE Marking Scheme, 2014] 
Ans.6 Any three of the following :
Sugarcane grows better in the south because it is a tropical crop and because of the black soil and
more scientific methods of cultivation.
The sugar mills are near the plantations, so there is no loss of sucrose.
This also lowers the transport costs.
The sugar industry is better managed in the south.
The factories are near the centres of consumption. [ICSE Marking Scheme, 2014] 
Ans.7 1. The cotton textile industry contributes nearly 30% of the value of exports and employs more than
55 million people.
2.  The industry is one of the largest foreign exchange earner commodities of India.
3. It contributes about 14% to the industrial production, 4% to the GDP and 14.42% to the country’s
export earnings.
Ans.8 (i) Maharashtra and Gujarat.
(ii) (a)  Regular supply or proximity to raw material.
(b) Favourable climatic conditions specially the humid climate.
(c) Good network of road and rail transportation within the country and sea routes for the
international market.

P-62 G E OG R A P H Y - X
(d) Location of major ports facilitates the export and import facilities.
(e) Availability of cheap and skilled labours. [Any two points] 
Ans.9 (i) Two reasons for the importance of the cotton textile industry-
1. The cotton textile industry contributes nearly 30% of the value of exports and employs more than
55 million people.
2. The industry is one of the largest foreign exchange earner commodities of India.
(ii) One reason for its poor performance-
1. Many factories are old, obsolete and sick industrial units and thus faces low productivity.
ll
 WORKSHEET-46
Ans.1 Bangalore.
Ans.2 Cotton grows widely in Maharashtra due to ideal climatic conditions, i.e. 21°C to 30°C of temperature,
50 to 75 cm of rain and black soil. 
Ans.3 Uttar Pradesh in North India and Tamil Nadu in South India. 
Ans.4 (i) Cotton in Ahmedabad
(ii) Silk in Mysuru 
Ans.5 (i) It is so because after harvesting the sucrose content in the cane begins to decrease.
(ii) Favourable climatic conditions with temperatures between 16°C and 32°C which is ideal for rearing
the silkworms. 
Ans.6 Molasses- It is thick, dark brown juice obtained from raw sugar during the refining process.
Bagasse- It is the dry pulpy residue left after the extraction of juice from sugarcane. 
Ans.7 The rearing of silk worms for the production of silk is known as Sericulture.
The two types of silk are- Mulberry and Tasar. 
Ans.8 The silk industry considered as a small scale industry because-
(a) Low capital investment i.e. less than one crore.
(b) It is a farm based and labour intensive industry and requires less machinery.
Two types of silk produced in India- Mulberry and Tasar. 
Ans.9 Bengaluru (Bengalore) and Mysuru (Mysore). 
Ans.10 (i) The Indian silk industry is an integral part of Indian Textile Industry and is one of the largest producers
of silk in the world.
(ii) The silk industry in India employs 60 lakh workers. 
Ans.11 Two reasons for the concentration of the sugar industry in Uttar Pradesh-
(i) Availability of fertile alluvial soil in the Ganga-Yamuna doab area.
(ii) Provides cheap labour as it is densely populated. 
Ans.12 (i) H
andloom industry is one of the oldest industries of India providing employment to millions of
skilled and unskilled people.
(ii) The handloom industry is mainly located in small town and rural areas and thus preserve our heritage
and culture.
(iii) They earn valuable foreign exchange. 
[Any two points]
Ans.13 Two economic advantages of the Handloom Industry-
(i) It provides employment.
(ii) It earns foreign exchange. 
Ans.14 (i) Kolkata has many cotton mills though cotton is not grown in West Bengal because :
(a) Kolkata is located very close to coal mines.
(b) It has abundant capital supply.

S O L U T I ON S P-63
(c) There is availability of cheap labour, humid climate, plenty of soft water.
(d) It has port facility and excellent means of transport and communication. [Any one point]
(ii) (a)  Karnataka has favourable climatic conditions with temperature ranging from 16°C to 32°C.
(b) Sericulture provides employment to a large number of people. [Any one point]
(iii) (a) The sugarcane produced in India is of poor quality with low sucrose content.
(b) In rural areas, instead of sugar, gur and khandsari are in more demand. 
Ans.15 Two by-products of Sugarcane–
Molasses : It is used in the production of citric acid, chemicals, synthetic rubber and as fuel for mills.
Bagasse : It is used as a biofuel and in the manufacture of pulp and building materials.
Pressmud : It is used for making wax, carbon paper and shoe polish. 
Ans.16 (i) Regular supply or proximity to raw material.
(ii) Favourable climatic conditions specially the humid climate.
(iii) Good network of road and rail transportation within the country and sea routes for the international
market.
(iv) Availability of cheap and skilled labours.
Ans.17 (a)  Kolkata is located very close to coal mines.
(b) It has abundant capital supply.
(c) There is availability of cheap labour, humid climate, plenty of soft water.
(d) It has port facility and excellent means of transport and communication. [Any three points]
Ans.18 (i) It is so because after harvesting the sucrose content in the cane begins to decrease.
(ii) Four sugar milling centres in the northern plains are Saharanpur, Meerut, Muzaffarnagar and
Baghpat.
ll

P-64 G E OG R A P H Y - X
DRAMA
CHAPTER

MINERAL RESOURCES
1
12
 WORKSHEET-47
Ans.1 Vishakhapatnam has an important ship-building yard because Vishakhapatnam is a sea port, has the
facility of a dry dock, gets its iron and steel from VISL, power from the Nagarjunasagar Dam, labour
from Andhra Pradesh, Odisha (Orrisa) and Karnataka. [ICSE Marking Scheme, 2014] 
Ans.2 Mini steel plants uses electric furnaces while integrated steel plants use blast furnaces.
Ans.3 (i) German Firm, Krupps and Demag.
(ii) Iron ore from the reserves of Sundargarh and Keonjhar district of Odisha.
Coal is obtained from Jharia, Talcher and Korba fields.
Ans.4 (i) Tata Iron and Steel Plant (TISCO).
(ii) Mini Steel plants are smaller units, work through electric furnaces and mainly uses steel scrap and
sponge iron or pig iron as their raw material.
Mini steel plants have the following advantages-
(a) Scarp iron is used as raw material which is cheap and easily accessible.
(b) These plants can be built with less capital investment.
(c) It does not cause pollution since it runs on electric power. [Any one point] 
Ans.5 ‘The Iron and Steel industry constitutes the backbone of modern industrial economy’ due to the
following reasons-
(i) It provides raw material to industries for manufacturing industrial machinery, railway engines,
railway tracks, electrical machinery, defence equipments, bridges, dams, shops, automobiles, etc.
(ii) India has one of the richest reserves of all the raw materials required for the industry, namely, capital,
cheap labour, iron ore, power, coal, etc.
Ans.6 (i) Rourkela Iron and Steel Plant.
(ii) Iron ore from the reserves of Sundargarh and Keonjhar district of Odisha. Manganese is obtained
from Barajmda.
Ans.7 It uses scrap iron / uses electric furnace so no pollution / conserves coal / help in decentralized
development. [ICSE Marking Scheme, 2016] 
Ans.8 The iron and steel plant at Durgapur was established with British collaboration.
Iron ore – Singbhum in Jharkhand / Keonjhar in Odisha
Manganese – Keonjhar in Odisha
Coal – Jharia coalfield in Jharkhand / Raniganj in West Bengal [ICSE Marking Scheme, 2016] 
Ans.9 (i) Iron and Steel
(ii) Electric locomotives
(iii) Aero-engines [ICSE Marking Scheme, 2016] 
Ans.10 (i) Engines for MIG aircraft - Koraput in Odisha (Orissa)
(ii) Diesel locomotives - Varanasi in Uttar Pradesh
(iii) Software - Bengaluru (Bangalore)/ Hyderabad / Pune. [ICSE Marking Scheme, 2014] 
Ans.11 (i) 1.  It does not cause pollution and the cost of production is low.
2.  Mini steel plant uses scrap iron, sponge iron, etc. as raw material from the large steel plants.
(ii) Iron Ore from Dalli Rajhara mines
Coal is obtained from Bokaro, Kargati and Jharia fields in Jharkhand and Korba in Chhattisgarh.
ll
 WORKSHEET-48
Ans.1 Bhilai Steel Plant.

S O L U T I ON S P-65
Ans.2 Rourkela Steel Plant.
Ans.3 Germany.
Ans.4 (i) It is called a basic industry because it supplies raw materials to all other industries. / Key product for
making of all industries/ used for automobiles/ railways / shipbuilding / heavy machineries / tools /
bridges/ roads/ domestic purpose etc
(ii) A mini steel plant is one which manufactures steel using scrap iron in electric arc furnaces.
[ICSE Marking Scheme, 2015] 
Ans.5 (i) Kanpur- aircraft/sugar/jute/electronic goods/automobiles(scooters) Leather industries.
[Any one point]
(ii) Rourkela- iron and steel/fertilizers. [Any one point]
(iii) Pune- automobiles/ electronics/ cotton textile.  [Any one point]
(iv) Mangalore- oil refining [ICSE Marking Scheme, 2015] 
Ans.6 Tata Iron and Steel Company
Iron - Singhbhum in Jharkhand / Mayurbhanj in Odisha (Orissa) / Bonai in Odisha (Orissa)
Coal - Jharia in Jharkhand / Bokaro in Jharkhand. [ICSE Marking Scheme, 2014] 
Ans.7 (a) Iron Ore from Dalli Rajhara mines
(b) Coal is obtained from Bokaro, Kargati and Jharia fields in Jharkhand and Korba in Chhattisgarh.
(c) Limestone developed in Nandini near Bhilai.
(d) Manganese obtained from neighbouring district of Balaghat. 
Ans.8 (i) Availability of iron, coal, manganese, limestone.
(ii) Supply of cheap labour from the states of Bihar and West Bengal. 
Ans.9 (i) It is so because it lacks the basic raw material required for iron and steel industry.
(ii) Iron and Steel industries require large land area, huge capital investments, modern technical
knowledge, huge machineries, etc.
Ans.10 (i) 1.  M
 ini Steel plants are smaller units than Iron and steel Industry.
2. It works through electric furnaces.
3. It mainly uses steel scrap and sponge iron or pig iron as their raw material.
(ii) The integrated steel plant at Jamshedpur gets its-
Iron ore from Gurumahisani mines in Mayurbhanj district of Odisha and
Noamundi mines in Singhbhum district of Jharkhand.
Coal from the Jharia and Bokaro coalfields. 
Ans.11 (i) A unified steel mill where all the primary functions of producing steel is carried out like iron making,
steel making, casting, roughing rolling and product rolling.
(ii) Bhilai Steel Plant.
Iron Ore from Dalli Rajhara mines
Coal is obtained from Bokaro, Kargati and Jharia fields in Jharkhand and Korba in Chhattisgarh.
ll

 WORKSHEET-49
Ans.1 Bengaluru and Hyderabad.
Ans.2 Pune- Electronics Industry.
Ans.3 Organic chemicals like naphtha, ethylene and benzene.
The raw materials are easily available since they are not agro-based Replacing traditional raw
materials like wood and metal
Cheap
Durable. [ICSE Marking Scheme, 2014] 
Ans.4 (i) (a) India has been experiencing a strong growth in the demand of consumer products and durables
in recent years which have facilitated the growth in the electronics, especially in the telecom sector
both directly and indirectly.

P-66 G E OG R A P H Y - X
(b) Indian electronic industry plays an important role in space technology and has launched many
indigenous satellites like APPLE and INSAT-1 and INSAT -2 Series.
(ii) Bengaluru and Hyderabad.
Ans.5 Chemical obtained either directly or indirectly from chemical processing, of petroleum oil or natural
gas is called Petrochemicals.
Two products made from petrochemicals are dyes, synthetic fibre, synthetic rubber, detergents,
insecticides and pesticides. [Any two] 
Ans.6 Two geographical reasons for the growth of IT industries in Bangalore-
(i) Cool and moderate climate neither cold nor warm.
(ii) Abundant supply of electricity through the hydropower projects.
(iii) Bengaluru (Bangalore) is well connected to other major metropolises of Madras and Hyderabad.
 [Any two]  
Ans.7 Bengaluru (Bangalore) is known as the electronic capital of India.
Two major centres of electronic products- Hyderabad, Bengaluru (Bangalore), Delhi, Pune, Mumbai,
etc.
Ans.8 (i) Natural gas and Crude oils are the two raw materials required by the petrochemical industry.
(ii) Two products are Adhesives, fertilizers, dyes, synthetic fibre, synthetic rubber, detergents, insecticides
and pesticides.
(iii) The petrochemical products are gaining importance because :
(a) Petrochemicals are cost effective, economic and cheap.
(b) The raw material is easily available and not depended on agricultural raw material.
Ans.9 (i) Petrochemical Industry : Adhesives and Fertilizers.
(ii) Heavy Engineering Industry : Not in Syllabus.
(iii) Electronic Industry : Computers and Television.
Ans.10 (i) This is because products made from petrochemicals are cheaper and more durable
(ii) A heavy engineering industry requires a huge capital investment because it requires a big plot of
land, a big labour force, consumes power and the raw material is heavy and expensive
(iii) The electronic industry is a fast developing industry because electronics are used in many fields,
computers, entertainment, medicine, engineering and education which is important for the
development of a country. [ICSE Marking Scheme, 2016] 
ll
 WORKSHEET-50
Ans.1 The electronic industry has had an effect both on entertainment because electronics are used
in televisions, video players, tape recorders, audio systems Education has been affected because
electronics are used computers, LCD projectors and software has been prepared to be used as
teaching aids. [ICSE Marking Scheme, 2014] 
Ans.2 Two petrochemical units are-
1. Herdillia Chemicals Ltd. in Chennai.
2. National Organic Chemicals Industries Ltd. near Mumbai.
3. Petrofils Cooperative Limited (PCL) in Vadodara.
4. Indian Petrochemical Corporation Ltd. near Vadodara. [Any two units] 
Ans.3 Most of the petrochemical units are in Maharashtra and Gujarat because-
(i) Maharashtra and Gujarat have oil refineries which can supply the basic raw materials of Naphtha or
Ethylene and Benzene to the petrochemical industry.
(ii) Both the states have excellent port facilities for import and export.
Ans.4 Petro-chemical are those chemicals obtained either directly or indirectly from chemical processing, of
petroleum oil or natural gas e.g. dyes, synthetic fibre, synthetic rubber, detergents, insecticides and
pesticides.
Petrochemicals are
1. cost effective
2. economic and cheap.

S O L U T I ON S P-67
Ans.5 Four petro-chemical products-
(i) Synthetic fibre
(ii) Synthetic rubber
(iii) Detergents
(iv) Insecticides.
Ans.6 (i) In defence electronics are used for making remote weapon, telecommunication, long distance
transmissions, etc.
(ii) Cool and moderate climate neither cold nor warm.
Ans.7 (a) Space Technology : Indian electronic industry plays an important role in space technology and has
launched many indigenous satellites like APPLE and INSAT-1 and INSAT -2 Series.
(b) Entertainment : Electronics are used in televisions and video players, computers and LCD
projectors. 
Ans.8 Two centres- Vadodara and Chennai.
Two products- Adhesives and fertilizers.
Ans.9 (i) Agriculture : Chemical industry provides fertilizers.
(ii) Public Health : Chemicals also produce medicines and vaccines for the improvement of health.
Ans.10 (i) This is because petrochemicals are cheaper/ durable, are not dependent on agricultural raw materials/
hence no fluctuation in production/ cost effective / raw materials easily available.  [Any two points]
(ii) Two products are- Plastic, synthetic rubber, PVC pipes, synthetic fibre, detergents.
[Any two points] 
[ICSE Marking Scheme, 2015]
Ans.11 (i) Heavy engineering industries require large amounts of power, labour intensive, High capital/bulky
raw materials/ High transport cost.  [Any two points]
(ii) Ship building yard on the east coast- Vishakhapatnam or Kolkata; electric locomotive- Chittaranjan.
[ICSE Marking Scheme, 2015] 
Ans.12 (i) Not in Syllabus
(ii) Bengaluru
(iii) Haldia Petrochemical Ltd. in West Bengal.
ll

P-68 G E OG R A P H Y - X
DRAMA
CHAPTER

TRANSPORT
1
13
 WORKSHEET-51
Ans.1 Advantage of the GQ project-Connects major cities and ports/provides an impetus to truck transport/
enables the industrial growth of the small towns through which it passes/provides opportunities for
the transport of agricultural produce from the hinterland to major cities and ports.
[Any one point] 
[ICSE Marking Scheme, 2015]
Ans.2 (i) Roadways are ideal for short distances as through it every village and hamlet can be reached.
(ii) It transports people and goods quickly and easily.
(iii) It provides door-to-door services. [Any two points] 
Ans.3 Railways can carry more tonnage and passengers than airways/facilitates easier movement of bulky
goods/helped to transfer new ideas/innovations to rural areas/they provide a more comfortable
journey especially for elders and infants/ Inexpensive/ contributes to the economic growth and
urbanisation.  [Any two points] 
[ICSE Marking Scheme, 2015]
Ans.4 Any two reasons:
– It is more suitable for short distance travel
– It provides door to door services
– Perishable products can be transported easily.
– Saves on cost of transporting from the station. [ICSE Marking Scheme, 2014] 
Ans.5 (i) Rail transport is cheap and helps in reaching the far and inaccessible areas.
(ii) 1. Passengers are travelling without tickets and sitting on the roof of the trains.
2.  Rise in rail accidents.
3.  The railway requires replacing the old rail tracks.
ll
 WORKSHEET-52
Ans.1 The North-South Corridor would connect Srinagar in Jammu and Kashmir with Kanyakumari in
Tamil Nadu.
Ans.2 (i) There are no traffic lights on expressways, unlike on highways/ two- wheelers and three-wheelers are
not permitted on expressways / there are no speed-breakers on expressways.  [Any two points]
(ii) The first Expressway-Ahmedabad-Vadodara Expressway [ICSE Marking Scheme, 2015]
Ans.3 Road is favoured in the northern plains because-
(i) It is a region of flat level land.
(ii) The cost for constructing rail tracks is low.
(iii) The northern region is a densely populated area.
(iv) The northern plains are rich in agricultural resources. [Any two points] 
Ans.4 (i) Railway lines cannot be constructed everywhere specially in the hilly regions and remote dense
forested regions.
(ii) Railway transport is unsuitable and uneconomical for short distance and small traffic of goods.
Ans.5 Golden Quadrilateral is the largest Express Highway project in India that connects India’s four biggest
metropolitan cities-Delhi, Mumbai, Kolkata and Chennai.
Two ways in which it will help in the economic development of the country-
(i) Establishing faster transport networks between major cities and ports.
(ii) Providing fast and smooth movement of products and people within India.
(iii) Developing industries and creating job opportunities in smaller towns through access to markets.
ll

S O L U T I ON S P-69
 WORKSHEET-53
Ans.1 North rivers flow over flat terrain, whereas Deccan rivers flow over uneven land, hence presence of
cataracts and rapids which hinder navigation/northern rivers are perennial, whereas Deccan rivers
are seasonal. [Any one point]
[ICSE Marking Scheme, 2015]
Ans.2 (i) Inland Waterway is declining because water transport is much slower than its roads, rail or air
transport.
(ii) Area of water transport is restricted. It provide access to limited areas.
Ans.3 (i) Air transport is expensive.
(ii) It connects only major cities.
(iii) It is dependent on weather conditions and can get delayed or cancelled causing inconvenience to
passengers. [Any one point]
It is still popular means of transportation because-
(i) Air transport is the fastest and most comfortable mode of transport.
(ii) It can easily reach to remote and inaccessible areas like mountains, forests, deserts etc. 
Ans.4 Helicopter provides services to the hilly regions, small islands, regions where rivers are flowing, dense
forest and remote areas.
Ans.5 Any one of the following:
Advantage :
Cheapest means of transport
Suitable for heavy and bulky goods
Fuel efficient
Eco-friendly
No traffic congestion
Comfortable with space for passengers
Any one of the following:
Disadvantage:
Depends on weather conditions
Slow means of transport
Can saw sea sickness
Restricted means of transport, can only be possible where rivers have to be navigable.
[ICSE Marking Scheme, 2014] 
Ans.6 (i) 1. The rivers are rain-fed and seasonal.
2. They are shorter.
3. These rivers have a number of waterfalls. [Any two points]
(ii) Chennai and Vishakhapatnam.
Ans.7 (i) National Waterway No.2 (NW-2)-The river Brahmaputra connects Dhubri-Pandu-Tezpur-Neamati-
Dibrugarh-Sadiya and connects the North East region with Kolkata and Haldia Ports.
(ii) Haldia.
ll

P-70 G E OG R A P H Y - X
DRAMA
CHAPTER

WASTE MANAGEMENT
1
14
 WORKSHEET-54
Ans.1 It is the process by which a body of water becomes enriched in dissolved nutrients that stimulate the
growth of aquatic plant life usually resulting in the depletion of dissolved oxygen.
Ans.2 Becomes an eyesore for spoils the landscape
Causes pollution
Affects terrestrial life adversely
Leads to health hazards. [ICSE Marking Scheme, 2014]
Ans.3 (i) Due to Methyl Isocynate Gas
(ii) Due to Methyl Mercury
(iii) Due to Nuclear Radiation.
ll
 WORKSHEET-55
Ans.1 Sewage is a type of wastewater either domestic or municipal wastewater that is produced from a
community of people e.g. wastewater from sinks, cloth washers, water used to flush toilets, soap
water, etc.
Ans.2 Waste management is a important because-
(i) Improperly stored refuse can cause health, safety and economic problems.
(ii) Transmission of diseases due to accumulation of wastes is a major threat to people and
environment.
Ans.3 The three sources of Methane gas are-
(i) Mining
(ii) Wetlands
(iii) Landfills
Ans.4 Acid rain is the result of excessive acids in rainwater which are formed when oxides like nitrogen
dioxide and sulphur dioxide react with water.
Any two of the following:
Increases acidity in the soil and destroys forests and crops
Corrodes buildings, monuments, bridges
Contaminates air and water
Affects the nervous system and health in general
Aquatic life affected adversely. [ICSE Marking Scheme, 2014]
ll
 WORKSHEET-56
Ans.1 One source of gaseous waste is automobiles, factories, burning of fossil fuels. [Any one]
Ans.2 The main objective is to minimise the generation of gaseous waste.
Ans.3 Asthma and Bronchitis.
Ans. 4 (i) Domestic waste
Industrial waste
Agricultural waste
Municipal waste  [Any two]
(ii) It is a waste which decompose through the actions of bacteria, fungi and other living organisms.
[ICSE Marking Scheme, 2016]

S O L U T I ON S P-71
Ans.5 (i) Waste should be segregated to make for easier waste management/ to help identify the best method
of waste disposal. [Any one point]
(ii) Nuclear waste is hazardous as it can remain radio-active for thousands of years/ if it is not disposed
off properly it continues to be hazardous/it can enter the human body food and water/ the damage it
causes is irreparable/ it affects future generations. [Any one point]
[ICSE Marking Scheme, 2015]
Ans.6 (i) Biodegradable waste is waste that can be easily broken down by natural processes of decomposition.
(ii) Exhaust from vehicles/burning of fossil fuels in factories and thermal power plants/burning of wheat
or rice straw/methane from cattle sheds.  [Any one point]
[ICSE Marking Scheme, 2015]
Ans.7 (i) Collection of Municipal Solid Wastes- By organising house-to-house collection of solid wastes and
keeping bio-medical and industrial wastes separate.
(ii) Storage of Municipal Solid Wastes- Storage facilities or bins should have easy to operate design for
handling, transfer and transportation of waste.
(iii) Transportation of Municipal Solid Wastes- Covered vehicles to prevent the wastes from being
scattered.
(iv) Segregation of Municipal Solid Wastes- Undertake phase programme to ensure community
participation in waste segregation. [Any three]
ll
 WORKSHEET-57
Ans. (i) Composting of waste is an aerobic method of decomposing solid wastes.
(ii) Incineration – It is the method of burning the waste to reduce its weight and volume so that it can be
disposed easily.
(iii) Segregation – It refers to separation of waste into different categories of waste. The most popular
segregation is biodegradable and non-biodegradable. [ICSE Marking Scheme, 2016]
ll
 WORKSHEET-58
Ans.1 The process of changing the waste and non-usable materials into potentially useful materials is called
Recycling. 
Ans.2 Learn to reduce consumption of goods/learn to recycle and reuse.  [Any one point]
[ICSE Marking Scheme, 2015]
ll

P-72 G E OG R A P H Y - X

You might also like